1filedownload.com · Web view2014/12/19  · Von-Hippel Lindau disease is a rare inherited...

159
1 A 61-year-old male who is a keen gardener presents with a lesion on his lower lip which bleeds intermittently for several months. On examination, it is a red area with thickened skin. Further examination reveals enlarged, non-tender lymph nodes in the anterior cervical chain. A punch biopsy confirms a squamous cell carcinoma. What is the most likely first step in the management of lymph node metastasis? Select one answer only Chemotherapy Chemoradiotherapy Radical neck dissection « CORRECT ANSWER Radiotherapy « YOUR ANSWER Wide local excision YOUR ANSWER WAS INCORRECT The Answer Comment on this Question Squamous cell carcinoma (SCC) of the mouth is related to betel nut chewing, common in Asia. Lymph node metastasis is treated with radical neck dissection, usually followed by radiotherapy. Similarly primary excision is usually followed by radiotherapy. Leucoplakia is a risk factor, especially if associated with severe epithelial dysplasia. Due to the genetic basis of cancer, anyone is at risk; however it is rare in non-smokers. 2

Transcript of 1filedownload.com · Web view2014/12/19  · Von-Hippel Lindau disease is a rare inherited...

Page 1: 1filedownload.com · Web view2014/12/19  · Von-Hippel Lindau disease is a rare inherited pathology that causes cyst development in multiple organs, including the pancreas. 52 A

1

A 61-year-old male who is a keen gardener presents with a lesion on his lower lip which bleeds intermittently for several months. On examination, it is a red area with thickened skin. Further examination reveals enlarged, non-tender lymph nodes in the anterior cervical chain. A punch biopsy confirms a squamous cell carcinoma.What is the most likely first step in the management of lymph node metastasis?

Select one answer only

Chemotherapy

Chemoradiotherapy

Radical neck dissection « CORRECT ANSWERRadiotherapy « YOUR ANSWERWide local excision

YOUR ANSWER WAS INCORRECT The Answer Comment on this Question

Squamous cell carcinoma (SCC) of the mouth is related to betel nut chewing, common in Asia. Lymph node metastasis is treated with radical neck dissection, usually followed by radiotherapy. Similarly primary excision is usually followed by radiotherapy. Leucoplakia is a risk factor, especially if associated with severe epithelial dysplasia. Due to the genetic basis of cancer, anyone is at risk; however it is rare in non-smokers.

2

Children’s bones are more elastic than adult bone.Which particular type of fracture affects elastic bone?

Colles’ fracture

Galeazzi fracture

Monteggia fracture

Tibial tubercle apophysitis

Page 2: 1filedownload.com · Web view2014/12/19  · Von-Hippel Lindau disease is a rare inherited pathology that causes cyst development in multiple organs, including the pancreas. 52 A

Torus and green stick fractures « YOUR ANSWER

YOUR ANSWER WAS CORRECT The Answer Comment on this Question

The skull and long bones in young children are still ossifying and therefore more malleable than in adults. Fractures to these areas in young children should therefore be treated with suspicion and no accidental injury (NAI) considered. Colles’ fractures are classical to, although not exclusive to, osteoporosis and therefore elderly adults.

3

A patient with a pleomorphic adenoma of the left parotid gland requires surgical resection. You are consenting the patient to surgery. 

What is the most common complication to warn him about?

A high risk of non-compressible bleeding

A post-operative sialocele (salivary fistula)

Loss of taste over the anterior two thirds of the tongue

Permanent facial weakness « YOUR ANSWERRedness and sweating in the cheek « CORRECT ANSWER

YOUR ANSWER WAS INCORRECT The Answer Comment on this Question

The axons conveying taste over the anterior two-thirds of the tongue and secretomotor fibres to the submandibular and sublingual salivary glands (the parotid is innervated by the IXth cranial nerve) are found in the chorda tympani nerve. The chorda tympani courses across the tympanic membrane beneath the mucous membrane of the middle ear after leaving the facial canal approximately 0.5 cm above the stylomastoid foramen. It exits the middle ear by passing through the petrotympanic fissure medially, and runs forwards on the medial side of the spine of the sphenoid bone. After entering the infratemporal fossa it merges with the lingual nerve. The chorda tympani, therefore, is not related to

Page 3: 1filedownload.com · Web view2014/12/19  · Von-Hippel Lindau disease is a rare inherited pathology that causes cyst development in multiple organs, including the pancreas. 52 A

the parotid gland.

The facial nerve runs in close proximity to the parotid gland and is easily damaged during surgery. In the context of tumours the surgeon may decide to remove the nerve deliberately to reduce the risk of recurrence. After surgery facial weakness may be present but usually resolves with time. Frey’s syndrome is caused when parasympathetic innervation of the normally sympathetic branches of the trigeminal nerve occurs. In this condition patients often develop sweating on the side of the cheek at the anticipation of eating.

Whilst bleeding and post operative sialocele are recognized complications of parotid surgery their risk can be minimized by good surgical techniques.

4

An 82-year-old man undergoes a left carotid endarterectomy under general anaesthetic. The operation is noted to be very difficult due to the high bifurcation of the common carotid artery. Post-operatively has speech is difficult to understand, and it is noted on protruding his tongue, it deviates to the left.Which nerve is most likely to be affected?

Ansacervicalis

Facial

Glossopharyngeal

Hypoglossal « YOUR ANSWERSpinal accessory

YOUR ANSWER WAS CORRECT The Answer Comment on this Question

The carotid sheath contains neurovascular structures and is situated between the vertebral and visceral compartments of the neck. The sheath contains the common carotid artery, which branches into the internal and external carotid arteries at the upper border of the thyroid cartilage. The internal carotid artery has no branches in the neck, whereas the external carotid artery has six branches in the neck. The carotid body is a chemoreceptor located at the bifurcation of the common carotid artery that monitors arterial levels of

Page 4: 1filedownload.com · Web view2014/12/19  · Von-Hippel Lindau disease is a rare inherited pathology that causes cyst development in multiple organs, including the pancreas. 52 A

oxygen and carbon dioxide. The vagus nerve courses through the length of the neck in the carotid sheath. The glossopharyngeal, accessory and hypoglossal nerves pass through the superior part of the sheath in their course to the pharynx, posterior triangle and tongue respectively.The hypoglossal nerve supplies all of the muscles of the tongue except palatoglossus, which is supplied by the pharyngeal plexus and accessory nerve. Due to it’s insertion on the posterolateral aspect of the tongue, if unopposed palatoglossus will deviate the tongue laterally.

5

An 85-year-old man with prostatic cancer is most likely to have metastatic spread of cancer through which of the following veins?

Single best answer question – choose ONE true option onlyTesticular vein

Cephalic vein

Basilic vein

External iliac vein

Internal vertebral venous plexus « YOUR ANSWER

YOUR ANSWER WAS CORRECT The Answer Comment on this Question

The veins of the internal vertebral venous plexus are clinically significant because they are valveless and can serve as a route for metastases. Cancerous cells can travel freely in vertebral veins and lodge somewhere else in the body. The other veins all have valves that would direct the flow of blood and stop some of the metastatic spread.

6

The application of arterial clips during total thyroidectomy is most likely to injure which of the following nerves?

Page 5: 1filedownload.com · Web view2014/12/19  · Von-Hippel Lindau disease is a rare inherited pathology that causes cyst development in multiple organs, including the pancreas. 52 A

External laryngeal « CORRECT ANSWERHypoglossal

Internal laryngeal « YOUR ANSWERPhrenic

Transverse cervical

YOUR ANSWER WAS INCORRECT The Answer

When the superior thyroid arteries are ligated, the external laryngeal nerves running alongside can be easily damaged. When the inferior thyroid arteries are ligated, the recurrent laryngeal nerves are also vulnerable. The internal laryngeal nerves pass above and behind the root of the superior thyroid arteries and are therefore usually outside the operative field. The hypoglossal nerves run in the anterior triangle of the neck superior to the hyoid bone and so are not in proximity. The phrenic nerves are protected as they lie behind the prevertebral fascia. Both transverse cervical nerves run in the subcutaneous fascia. Horizontal skin-crease thyroidectomy incisions run parallel with their course, and therefore most branches of these cutaneous nerves are spared.

7

A 72-year-old diabetic male who is a lifelong smoker is referred by his GP to a rapid access TIA clinic because of episodes of transient vision loss affecting his left eye. In the clinic he is thought to have had episodes of amaurosisfugax due to emboli affecting the ophthalmic artery.The ophthalmic artery is a branch of which vessel?

Select one answer only

Anterior cerebral artery

External carotid artery « YOUR ANSWERFacial artery

Page 6: 1filedownload.com · Web view2014/12/19  · Von-Hippel Lindau disease is a rare inherited pathology that causes cyst development in multiple organs, including the pancreas. 52 A

Internal carotid artery « CORRECT ANSWERMiddle cerebral artery

YOUR ANSWER WAS INCORRECT The Answer

The ophthalmic artery is a branch of the internal carotid artery. It passes through the optic canal and supplies the ethmoidal air cells, part of the lateral wall of the nose, external nose, eyelids and forehead. It also supplies all the muscles of the orbit.

8

A patient who had surgery in the left carotid triangle complained to his physician that he has little sense of touch to the skin over the left side of his neck and difficulty swallowing. The patient’s hyoid bone is deviated to the right side. The patient’s tongue is not affected. The physician suspects that the cervical plexus of nerves to the left side of this patient’s neck was harmed during the surgical procedure.Of the following nerves, which is embedded in the carotid sheath and therefore vulnerable to injury during surgical procedures to the carotid artery?

Single best answer question – choose ONE true option onlySpinal accessory nerve « YOUR ANSWERAnsa cervicalis « CORRECT ANSWERCervical sympathetic chain

Phrenic nerve

Suprascapular nerve

YOUR ANSWER WAS INCORRECT The Answer Comment on this Question

The ansa cervicalis (or ansa hypoglossi in older literature) is a loop of nerves that are part of the cervical plexus.

Page 7: 1filedownload.com · Web view2014/12/19  · Von-Hippel Lindau disease is a rare inherited pathology that causes cyst development in multiple organs, including the pancreas. 52 A

Branches from the ansa cervicalis innervate the sternohyoid, sternothyroid and the inferior belly of the omohyoid. Two roots make up the ansa cervicalis. The superior root of the ansa cervicalis is formed by a branch of spinal nerve C1. These nerve fibres travel in the hypoglossal nerve before leaving to form the superior root. The superior root goes around the occipital artery and then descends embedded in the carotid sheath. It sends a branch off to the superior belly of the omohyoid muscle and is then joined by the inferior root. The inferior root is formed by fibres from spinal nerves C2 and C3.

9

You are in the ENT clinic reviewing a patient with a midline swelling of the neck.

The differential diagnosis includes:Arteriovenous fistula

Branchial cyst

Chemodectoma

Cystic hygroma

Subhyoid bursa « YOUR ANSWER

YOUR ANSWER WAS CORRECT The Answer Comment on this Question

Swellings of the neck in the midline include 1. thyroglossal cysts, 2. pharyngeal pouches, 3. plunging ranulae, 4. subhyoid bursae, 5. laryngoceles 6. and lesions in the thyroid isthmus.

10

Page 8: 1filedownload.com · Web view2014/12/19  · Von-Hippel Lindau disease is a rare inherited pathology that causes cyst development in multiple organs, including the pancreas. 52 A

You examine a patient’s cranial nerves in clinic, turning their head to the left against direct opposing pressure from your hand.In this motion which of the following statements is correct?

Select one answer only

Movement is limited by cervical vertebrae

Movement takes place at the atlanto-occipital joint

Neural impulses pass via the cranial accessory nerve

The axis of rotation runs vertically through the odontoid process « CORRECT ANSWER

The left sternocleidomastoid muscle is the main agonist « YOUR ANSWER

YOUR ANSWER WAS INCORRECT The Answer Comment on this Question

The head is turned using the contralateral sternocleidomastoid muscle, stimulated by impulses passing in the spinal accessory nerve. The cranial accessory nerve fibres join the vagus nerve soon after exiting the brain stem. The movement takes place at the atlantoaxial joint about a vertical axis through the odontoid process of the axis, and is limited by the vertical alignment of the two attachments of the contralateral sternocleidomastoid muscle (contralateral mastoid process and sternoclavicular joint).

11

You examine a patient in A&E who has sustained facial injuries in an RTA, he has mobility of the maxillary segment on examination, and the x-rays show a fracture line passing transversely across the maxillary sinus and pterygoid plates. 

This type of fracture is:

Le Fort 1 « YOUR ANSWERLe Fort 2

Le Fort 3

Page 9: 1filedownload.com · Web view2014/12/19  · Von-Hippel Lindau disease is a rare inherited pathology that causes cyst development in multiple organs, including the pancreas. 52 A

Le Fort 4

Craniofacial dysfunction

YOUR ANSWER WAS CORRECT The Answer Comment on this Question

The fractures of the maxilla can be classified into Le Fort fracture classes 1 to 3. Mobility of the maxillary segment is the pathognomonic sign of Le Fort fracture. Clinically, it is demonstrated by stabilising the glabella with one hand while distracting the alveolus with the other. In Le Fort 1 fracture, the fracture line passes transversely across the base of the piriform aperture, the base of the maxillary sinus and the pterygoid plates. 

Movement of the alveolus and nasofrontal region suggests a Le Fort 2 fracture; movement of the alveolus alone suggests Le Fort 1 fracture. In Le Fort 2 fracture, the bony fragment contains the lacrimal crests, the bulk of the maxilla, the piriform margin, the alveolus and the palate. Le Fort 3 fracture is associated with detachment of the entire midfacial skeleton from the cranial base. It is known as craniofacial dysfunction

12

You are performing a cranial nerve examination on a patient who has a mild head injury. You ask them to stick out their tongue.With regards to the examination of the tongue:

It is depressed by the hyoglossus « CORRECT ANSWERIt is passive during the voluntary phase of swallowing

It is protruded by the styloglossus

It is retracted by the hyoglossus muscle

It receives sensory innervation from the vagus nerve « YOUR ANSWER

YOUR ANSWER WAS INCORRECT The Answer Comment on this Question

Page 10: 1filedownload.com · Web view2014/12/19  · Von-Hippel Lindau disease is a rare inherited pathology that causes cyst development in multiple organs, including the pancreas. 52 A

The sensory innervation to the tongue is from the VIIth (facial) and IXth (glossopharyngeal) cranial nerves and the lingual nerve. The tongue deviates to the side of a XIIth cranial nerve lesion on protrusion, is active during the first stage of swallowing and contains the lingual tonsil in the dorsum of its posterior third. The tongue is retracted up and back by the styloglossus muscle, protruded by genioglossus and depressed by the hyoglossus.

13

You are in theatre performing a formal tracheostomy.

When performing this procedure:

It is best performed with the neck slightly flexed

Midline dissection provides a relatively avascular plane « CORRECT ANSWEROnce the tracheostomy cuff is inflated further fixation is not required « YOUR

ANSWERThe first and second tracheal rings are incised

The skin incision is placed between the cricoid cartilage and hyoid

YOUR ANSWER WAS INCORRECT The Answer Comment on this Question

Tracheostomy is a definitive surgical airway. The head and neck are extended, and a 3–4 cm transverse collar incision is made between the cricoid cartilage and sternal notch. Once the incision is deepened through the platysma muscle, the remainder of dissection should continue in the midline in a relatively avascular plane down to the pretracheal fascia. This layer is then incised to reveal the tracheal rings. Care must be taken not to incise the first ring (the cricoid cartilage). A vertical incision through the second and third rings is usual. Once the airway is in the trachea, the cuff may be inflated. Further fixation is then mandatory to prevent the tracheostomy tube from being displaced.

Page 11: 1filedownload.com · Web view2014/12/19  · Von-Hippel Lindau disease is a rare inherited pathology that causes cyst development in multiple organs, including the pancreas. 52 A

14

The olfactory foramina:

Single best answer question – choose ONE true option onlyAre located in the middle cranial fossa « YOUR ANSWERAre located in the anterior cranial fossa « CORRECT ANSWERAre located immediately inferior to the optic foramen

Have motor neurones running through them

Are located in the sphenoid bone

YOUR ANSWER WAS INCORRECT The Answer Comment on this Question

The olfactory foramina are located in the anterior cranial fossa. These foramina are in the cribriform plate of the ethmoid bone for the passage of olfactory nerves.

15

A 32-year-old builder presents with back and severe left-leg pain radiating to his foot after lifting a heavy box. MRI scan demonstrates an L5/S1 paracentral intervertebral disc bulge.With these observations and findings which of the following is most likely to be correct?

Select one answer only

An absent ankle jerk means that the disc should be removed

Numbness on the sole of the foot may be found on examination « YOUR ANSWER

Pain radiating to the buttocks would be due to nerve root involvement

The exiting rather than the transiting nerve root is most likely to be compressed at this level

Urinary retention would never be secondary to pain and bedrest

Page 12: 1filedownload.com · Web view2014/12/19  · Von-Hippel Lindau disease is a rare inherited pathology that causes cyst development in multiple organs, including the pancreas. 52 A

YOUR ANSWER WAS CORRECT The Answer

Nerve root involvement is suggested by pain radiating below the knee. It is worse when the nerve is stretched, as with the sciatic stretch test, or when intra-abdominal pressure is raised, as with straining at stool, coughing or sneezing. Urinary retention is a worrying sign but is not infrequently secondary to pain, analgesic medication and bedrest. Before this can be assumed, a full neurological examination must be performed, and if there are concerns regarding a caudaequina syndrome then further investigations must be carried out.Paracentral disc bulges are the most common in the lumbar region and usually impinge on the transiting rather than the exiting nerve root. In this case that wo uld be the S1 root which can result in

plantar flexion weakness, numbness on the sole of the foot in the S1 dermatome and an absent ankle jerk reflex.

An absent ankle jerkimplies some nerve root involvement, but on its own does not demand surgical intervention.

16

Which of the following structures is least likely to be encountered on left carotid endarterectomy?

Facial vein « YOUR ANSWERPleural membranes « CORRECT ANSWERSympathetic chain

Thoracic duct

Vagus nerve

YOUR ANSWER WAS INCORRECT The Answer Comment on this Question

The vagus nerve travels in the carotid sheath in the neck. The sympathetic trunk lies alongside the cervical vertebrae, immediately behind the carotid artery, and

Page 13: 1filedownload.com · Web view2014/12/19  · Von-Hippel Lindau disease is a rare inherited pathology that causes cyst development in multiple organs, including the pancreas. 52 A

has three cervical ganglia (superior, middle and inferior). During a left carotid endarterectomy (CEA), the thoracic duct may be seen. Pleural membranes are usually a little deeper and lateral to the site of CEA. The hypoglossal nerve passes forwards to supply the tongue and has to be protected, while the facial vein has to be ligated and divided to mobilise the internal jugular vein.

17

You are performing a parotidectomy.Which of the following lie within the gland you are excising?

External carotid artery « CORRECT ANSWERMasseter

Maxillary artery

Superficial temporal artery

Trigeminal nerve « YOUR ANSWER

YOUR ANSWER WAS INCORRECT The Answer

The parotid gland lies deep to masseter, medial pterygoid, stylomandibular ligament, superficial temporal artery, maxillary artery and facial nerve. Within the gland lie the

facial nerve, retromandibular vein, external carotid artery and auriculotemporal nerve.

18

A 78-year-old woman on home O2 for COPD has sustained a neck of femur fracture. The anaesthetist has decided a spinal anaesthetic will be safer.What vertebral level does the spinal cord end at in adults?

Page 14: 1filedownload.com · Web view2014/12/19  · Von-Hippel Lindau disease is a rare inherited pathology that causes cyst development in multiple organs, including the pancreas. 52 A

L1 - L2 « YOUR ANSWERL3 - L4

S1 - S2

T7 - T8

T11 - T12

YOUR ANSWER WAS CORRECT The Answer

The Angle of Louis is at the level of T4. The spinal dura mater ends at S2, it is the spinal cord that ends between L1 and L2. The oesophagus traverses the diaphragm at the level of T10, the aorta traverses the diaphragm at the level of T12.

19

Endocrine abnormalities can be associated with neurosurgical conditions. The hypthalmus and pituitary gland are the dominant controllers of the endocrine system.What structure lies lateral to the pituitary gland? 

Cavernous sinus « YOUR ANSWERDiaphragma sellae

Optic chiasm

Sphenoid sinus

Temporal lobe

YOUR ANSWER WAS CORRECT The Answer Comment on this Question

The pituitary sits in the sella turcica of the sphenoid bone. It is covered by the diaphragma sellae (fold of dura) which separates it from the optic chiasma above. There is an opening in the diaphragma for the pituitary stalk.

Page 15: 1filedownload.com · Web view2014/12/19  · Von-Hippel Lindau disease is a rare inherited pathology that causes cyst development in multiple organs, including the pancreas. 52 A

The cavernous sinuses are found laterally including

1. the internal carotid artery 2. and cranial nerves III, 3. IV, 4. Va, 5. Vb 6. and VI and the sphenoid sinuses inferiorly.

20

Which of the following structures separates the anterior and posterior chambers in the eyeball?

Single best answer question – choose ONE true option onlyThe lens

The cornea

The iris « YOUR ANSWERThe pupil

The ciliary processes

YOUR ANSWER WAS CORRECT The Answer

Page 16: 1filedownload.com · Web view2014/12/19  · Von-Hippel Lindau disease is a rare inherited pathology that causes cyst development in multiple organs, including the pancreas. 52 A

Comment on this QuestionThe iris has received its name from its various colours in different individuals. It is a thin, circular, contractile disc, suspended in the aqueous humour between the cornea and lens and perforated a little to the nasal side of its centre by a circular aperture, the pupil. At its periphery it is continuous with the ciliary body and is also connected to the posterior elastic lamina of the cornea by means of the pectinate ligament. Its surfaces are flattened and look forward and backward, the anterior toward the cornea, the posterior toward the ciliary processes and lens. The iris divides the space between the lens and the cornea into an anterior and a posterior chamber. The anterior chamber of the eye is bounded in front by the posterior surface of the cornea; behind by the front of the iris and the central part of the lens. The posterior chamber is a narrow chink behind the peripheral part of the iris and in front of the suspensory ligament of the lens and the ciliary processes. In the adult, the two chambers communicate through the pupil, but in the fetus up to the seventh month they are separated by the membrana pupillaris.

21

You are assisting your consultant in a parotidectomy for suspected cancer and he is asking you about the anatomical structures you encounter during the operation.The parotid gland:

Has a duct that passes posterior and deep to the masseter « YOUR ANSWERIs encapsulated by an investing layer of superficial cervical fascia

Is separated from the carotid sheath by the styloid process « CORRECT ANSWER

Is traversed by the facial artery

Receives its blood supply directly from the internal carotid artery

YOUR ANSWER WAS INCORRECT The Answer Comment on this Question

The fibrous capsule of the parotid gland is an upward extension of the deep investing layer of cervical fascia that attaches to the zygomatic arch. The medial wall of the capsule is separated from the carotid sheath by the styloid process

Page 17: 1filedownload.com · Web view2014/12/19  · Von-Hippel Lindau disease is a rare inherited pathology that causes cyst development in multiple organs, including the pancreas. 52 A

and associated muscles (stylopharyngeus, stylohyoid, styloglossus). The external carotid artery passes through the gland, supplying it as it does so. No facial vessels are related to the gland. The facial nerve enters the gland and divides within it into its five terminal branches. The duct passes anteriorly and superficial to the masseter.

22

A 28-year-old man presents with a septic cavernous sinus thrombosis, with high fever, orbital oedema and proptosis. The primary source of infection would most likely arise from which site?Single best answer question – choose ONE true option only  

The chin

The occipital region

The skin over the parotid gland « YOUR ANSWERThe pinna of the ear

The upper lip « CORRECT ANSWER

YOUR ANSWER WAS INCORRECT The Answer Comment on this Question

The cavernous sinus lies on either side of the body of the sphenoid. Anteriorly, the ophthalmic veins drain into the sinus and communicate with the anterior facial vein, which drains the face and upper lip – hence the danger of spread of infection from this locus.

23

During submandibular gland excision a branch of the facial nerve is thought to be injured.Which of the following manifestations is likely to occur?

Page 18: 1filedownload.com · Web view2014/12/19  · Von-Hippel Lindau disease is a rare inherited pathology that causes cyst development in multiple organs, including the pancreas. 52 A

Corneal ulceration

Drooping of the corner of the mouth « YOUR ANSWERDry mouth

Tongue ulceration

Weakness in tongue movements

YOUR ANSWER WAS CORRECT The Answer

Seventy-five per cent of cases of acute facial nerve palsy are of unknown aetiology (Bell’s palsy); hence computed tomography (CT) scanning is not usually required in the presence of few clinical signs. The House–Brackmann scale of I (normal) to IV (complete paralysis) gives an indication of severity, with eye closure the most important observation as this can lead to corneal ulceration (grade III and above). Marginal mandibular nerve damage in submandibular gland surgery manifests as drooping of the corner of the mouth. Palsy following superficial parotidectomy is in the region of 0.5%.

24

Which of the following types of nerve fibre does the spinothalamic tract carry? 

Descending motor fibres

First order fibres carrying light touch and proprioception

First order fibres carrying pain and temperature sensation

Second order fibres carrying light touch and proprioception « YOUR ANSWER

Second order fibres carrying pain and temperature sensation « CORRECT ANSWER

YOUR ANSWER WAS INCORRECT The Answer Comment on this Question

Page 19: 1filedownload.com · Web view2014/12/19  · Von-Hippel Lindau disease is a rare inherited pathology that causes cyst development in multiple organs, including the pancreas. 52 A

The spinothalamic tract conveys pain, temperature, touch and pressure sensations from one side of the body to the opposite side of the brain. Vibration and position sense are conveyed via the posterior column. The first neurone of the spinothalamic tract synapses in the posterior horn; the next neurone crosses to the right side of the spinal cord and synapse in the thalamus, after ascending through the cord and brainstem; the third neurone arises in the thalamus to pass to the cortex. The secondary axons of the spinothalamic tract ascend through the brainstem to synapse in the thalamus. Axons from the cervical region synapse medially while axons from the lumbar region synapse laterally. A lesion of the spinothalamic tract anywhere in the brainstem would lead to a loss of pain sensations from the opposite side of the body. Temperature and touch sensations would also be diminished from the opposite side of the body but not totally lost because other pathways may also convey these modalities. A lesion of the spinothalamic tract at the level of the spinal cord would lead to loss of pain sensations on the opposite side, beginning one level below the level of the lesion.

25

A 34-year-old man presents with C-spine tenderness after being involved in a road traffic accident. He reports swerving suddenly to avoid a child on the road and colliding with a parked car. He admits to not wearing a seat belt and injuring his chin as he fell forward into the dashboard with his neck hyperextended.Which fracture pattern is most likely to be seen on x-ray?

Hangman’s fracture « YOUR ANSWERJefferson fracture

Type I Odontoid fracture

Type II Odontoid fracture

Type III Odontoid fracture

YOUR ANSWER WAS CORRECT The Answer Comment on this Question

Page 20: 1filedownload.com · Web view2014/12/19  · Von-Hippel Lindau disease is a rare inherited pathology that causes cyst development in multiple organs, including the pancreas. 52 A

Acute fractures of the axis, (C2 vertebra), represent about 18% of all cervico-spinal injuries and approximately 60% of axis fractures involve the odontoid process. The odontoid process is a peg-shaped bony protuberance that projects upward and is normally positioned in contact with the anterior arch of C1. It is held in place primarily by the transverse ligament. Type I odontoid fractures involve the tip of the odontoid peg, type II fractures are through the base of the dens (involving the junction of the odontoid peg with the body) and type III fractures occur at the base of the dens and extend obliquely into the body of the axis. Odontoid fractures are initially identified by a lateral cervico-spinal film or open-mouth odontoid views. In many cases, however, a computed tomography (CT) scan is required to further delineate the type and extent of the fracture. In children younger than 6 years of age, on plain radiography, the epiphysis may be prominent and may look like a fracture at this level. Type II is the commonest type of odontoid fractures. They require surgical reduction and immobilisation with a Halo and body cast. If the fracture is not healed (and so unstable) at 12 weeks, posterior fusion of C1 to C2 may be indicated. The posterior elements of C2, ie, the pars interarticularis may be fractured, (a hangman’s fracture), by an extension type of injury. Patients with this fracture should be maintained in external immobilisation until specialised care is available. These fractures represent approximately 20% of all axis fractures.

26

A 67-year-old male suffers an embolic stroke. Duplex ultrasonography demonstrates a severe right common carotid artery stenosis and he proceeds to undergo a carotid endarterectomy.Which of the following best describes the right common carotid artery?

Select one answer only

It bifurcates at the level of the upper border of the cricoid cartilage

It has the cervical sympathetic chain as an anterior relation

Page 21: 1filedownload.com · Web view2014/12/19  · Von-Hippel Lindau disease is a rare inherited pathology that causes cyst development in multiple organs, including the pancreas. 52 A

It is a branch of the aortic arch

It is enclosed within the carotid sheath throughout its course « YOUR ANSWER

It lies lateral to the lateral lobe of the thyroid gland

YOUR ANSWER WAS CORRECT The Answer

The right common carotid artery branches off the brachiocephalic artery. It bifurcates at the level of the upper border of the lamina of the thyroid cartilage. It lies posterior to the lobes of the thyroid gland and anterior to both the cervical sympathetic chain and the phrenic nerve on the scalenus anterior muscle; the latter is separated from the artery by prevertebral fascia.

27

An anaesthetic ST4 was called to the resuscitation bay of the A&E department to help secure the airway of a child with breathing difficulties. After recognising that the child would need to be intubated shortly he called a senior colleague for support. 

Which of the following reasons make intubating children more difficult?

Although there are no anatomical differences between adults and children the fact that children are smaller makes intubation more difficult

Anatomical differences between adults and children such as larger head size, a U-shaped epiglottis and a relatively larger tongue make intubation more difficult« CORRECT ANSWER

Anatomical differences between adults and children such as a more caudally placed larynx and a smaller angle of the jaw make intubation more difficult

Intubating a child is no more difficult than intubating an adult, this trainee called a senior colleague because he was relatively junior and needed support« YOUR ANSWER

Situations involving children are usually more tense because family members are often present

Page 22: 1filedownload.com · Web view2014/12/19  · Von-Hippel Lindau disease is a rare inherited pathology that causes cyst development in multiple organs, including the pancreas. 52 A

YOUR ANSWER WAS INCORRECT The Answer Comment on this Question

Children have a relatively larger head, which tends to flex the head on the neck, making airway obstruction more likely. The relatively larger tongue tends to flop back and obstruct the airway in the obtunded child, which means that there is less room in the mouth when intubation is being carried out. The larynx is positioned more cephalic (glottis at C3 in infants compared with C6 in adults) and the angle of the jaw is larger in children (140º in infants, 120º in adults), both of which make intubation more difficult. In addition, the trachea is shorter and the cricoid ring is the narrowest part of the airway (compared with the glottis in the adult).

28

A 57-year-old manual labourer attends with a rough, scaly patch on his lower lip. On taking a history he reveals it bleeds when bumped and on examination a scaly, red patch with irregular borders is seen.What is the commonest carcinoma of the head and neck?

Select one answer only

Basal cell carcinoma

Keratoacanthoma

Melanoma

Merkel cell carcinoma

Squamous cell carcinoma « YOUR ANSWER

YOUR ANSWER WAS CORRECT The Answer Comment on this Question

Squamous-cell carcinoma (SCC) is the commonest carcinoma of the head and neck. Of the oral cavity, the lower lip is frequently implicated, probably due to prolonged exposure to sunlight. Leucoplakia is a premalignant condition and is usually painless, hence their relatively late presentation. Lymph-node involvement is common, reflecting the usual delay in presentation and indicates

Page 23: 1filedownload.com · Web view2014/12/19  · Von-Hippel Lindau disease is a rare inherited pathology that causes cyst development in multiple organs, including the pancreas. 52 A

a poorer prognosis. Seventy-five per cent of lingual cancers present on the anterior two-thirds.

29

You are assisting in a carotid endarterectomy and are asked about the anatomy of the internal carotid artery.Which statement is correct regarding the internal carotid artery?

It commences at the level of C6

It divides into the middle and posterior cerebral arteries

It gives off the ophthalmic artery « YOUR ANSWERIt has two extra-cranial branches

It passes through the foramen ovale

YOUR ANSWER WAS CORRECT The Answer Comment on this Question

The common carotid artery bifurcates into the external and internal carotids at the level of the upper part of the C4 vertebra ,ie the upper border of the thyroid cartilage, however this bifurcation is frequently higher, near the tip of the great horn of the hyoid bone (C3 level). The internal carotid artery has no extra cranial branches and enters the base of the skull in the petrous temporal bone through the carotid canal. The internal carotid on entering the skull passes forwards through the temporal bone upwards into the cavernous sinus, turns forward and upwards through the roof of the sinus to lie medial to the anterior clinoid process before turning back on itself above the cavernous sinus and then passing once more lateral to the optic chiasma to end by dividing into the anterior and middle cerebral arteries. The ophthalmic artery originates from the internal carotid artery immediately above the roof of the cavernous sinus.

30

Page 24: 1filedownload.com · Web view2014/12/19  · Von-Hippel Lindau disease is a rare inherited pathology that causes cyst development in multiple organs, including the pancreas. 52 A

A patient has her inferior laryngeal nerve inadvertently divided during a partial thyroidectomy.Which clinical features are likely to result from this?

Single best answer question – choose ONE true option only

The larynx is anaesthetised inferior to the vocal cord on the affected side « YOUR ANSWER

The larynx is totally anaesthetised on the affected side

All the laryngeal muscles on the affected side are paralysed

All the laryngeal muscles are paralysed on the affected side, apart from the posterior cricoarytenoid muscle

At laryngoscopy, the affected cord is seen to lie paralysed in the midline

YOUR ANSWER WAS CORRECT The Answer

The inferior or recurrent laryngeal branch of the vagus nerve (X) supplies motor fibres to all the muscles of the larynx apart from the cricothyroid muscle, and sensory fibres to the larynx inferior to the vocal cords. Injury to this nerve will result in paralysis of all muscles of the larynx except cricothyroid, and paralysis of the vocal cord. Paralysed vocal cords lie in the “paralytic position”, slightly abducted from the midline, and do not move with phonation. The sensory loss from this nerve injury will be inferior to the vocal cords on the affected side.

Cricothyroid is supplied by the superior laryngeal branch of the vagus nerve.

31

Which one of the following is true regarding the thyroid gland?Select one answer only

Derives its arterial supply solely from branches of the external carotid

Has a venous plexus draining into the internal jugular and brachiocephalic veins « CORRECT ANSWER

Page 25: 1filedownload.com · Web view2014/12/19  · Von-Hippel Lindau disease is a rare inherited pathology that causes cyst development in multiple organs, including the pancreas. 52 A

Initially moves down on swallowing « YOUR ANSWERLies superficial to the myofascial layer in the neck

Receives 20% of the total cardiac output

YOUR ANSWER WAS INCORRECT The Answer Comment on this Question

The thyroid gland lies deep to the myofascial layer (strap muscles and investing layer of deep cervical fascia), closely applied to the thyroid and cricoid cartilages. The gland initially moves up on swallowing before returning to its normal position. The thyroid gland is highly vascular, normally accounting for 5% of cardiac output.

While the superior thyroid artery is a branch of the external carotid artery, the inferior thyroid artery arises from the subclavian artery, via the

thyrocervical trunk. There may also be a ‘thyroideaima’ artery that may arise from the aortic

arch, the brachiocephalic artery or even the internal mammary artery. The thyroid venous plexus usually drains via three pairs of veins: the superior and middle thyroid veins drain into the internal jugular; the inferior thyroid veins drain into the brachiocephalic vein.

32

A 33-year-old male presents to his GP concerned about several 1mm- white spots around the vermillion border of his lips. On examination they are smooth and non-tender.Which one of the following is most likely the cause?

Bowen’s disease

Candidiasis « YOUR ANSWERFordyce’s granules « CORRECT ANSWERHairy leucoplakia

Speckled leucoplakia

Page 26: 1filedownload.com · Web view2014/12/19  · Von-Hippel Lindau disease is a rare inherited pathology that causes cyst development in multiple organs, including the pancreas. 52 A

YOUR ANSWER WAS INCORRECT The Answer Comment on this Question

Fordyce’s granules are creamy spots of sebaceous glands and are frequently mistaken for disease. Oral candida has a low malignant potential. True frictional keratosis has no malignant potential. Hairy leucoplakia strongly suggests HIV infection; if negative other rarer immunodeficiency syndromes should be considered.

33

The facial nerve is a structure of utmost importance in head and neck surgery.Which of the following muscles is supplied by the facial nerve? 

Anterior belly of digastric

Buccinator « CORRECT ANSWERMedial pterygoid

Mylohyoid

Temporalis « YOUR ANSWER

YOUR ANSWER WAS INCORRECT The Answer Comment on this Question

The buccal branch of the nerve pierces the buccinator muscle after supplying it and the muscles of the upper lip. The trigeminal (V) nerve is predominantly associated with the muscles of mastication. These include, masseter, temporalis and the pterygoid muscles. There are five main divisions of the facial nerve originating from the parotid gland and the lowest or fifth branch is the cervical. The facial nerve does pass through the stylomastoid foramen.

34

A 69-year-old man presents with recurrent episodes of epistaxis.

Page 27: 1filedownload.com · Web view2014/12/19  · Von-Hippel Lindau disease is a rare inherited pathology that causes cyst development in multiple organs, including the pancreas. 52 A

What is true of epistaxis?Single best answer - select one answer only

It is a common presentation of leukaemia

It most commonly originates from the lateral nasal wall

It has no proven association with hypertension « CORRECT ANSWERWhere caused by an inflammatory reaction it frequently requires treatment by

submucosal resection (SMR)

When caused by foreign bodies, the bleeding can usually be stopped by nasal packing with cotton wool« YOUR ANSWER

YOUR ANSWER WAS INCORRECT The Answer Comment on this Question

Epistaxis most commonly arises from the anterior part of the septum – Little’s area. Although epistaxis was traditionally associated with hypertension, this association has been disproved by various recent studies. More commonly, the anxiety associated with epistaxis produces a hypertensive picture in such patients. Treatment should be focused on controlling haemorrhage and reducing anxiety as primary means of blood pressure reduction. Treatment may require nasal/post-nasal packing. Nasal packing is performed using 1-inch ribbon gauze which may be soaked in 5% cocaine and adrenaline (1/1000). Surgical treatment is rarely necessary and SMR is used if bleeding is from behind a septal spur or if deviation prevents packing. 

35

A 22-year-old man sustained facial injuries playing rugby. 

Which radiological investigation is the most helpful to plan surgery?

Angiography

Computed tomography (CT) scanning « YOUR ANSWERFacial view X-rays showing fluid in the paranasal sinuses

MRI

Waters’ facial view

Page 28: 1filedownload.com · Web view2014/12/19  · Von-Hippel Lindau disease is a rare inherited pathology that causes cyst development in multiple organs, including the pancreas. 52 A

YOUR ANSWER WAS CORRECT The Answer Comment on this Question

Computed tomography (CT) scanning is essential when planning reconstruction of facial fractures, especially complex ones. CT scanning also delineates soft tissue damage adequately. Indirect signs of a fracture include soft tissue swelling, periorbital or intracranial air and fluid in the paranasal sinus. Four facial X-rays are usually taken: Waters’ view (posterior–anterior (PA) with cephalad angulation), Caldwell (PA view), lateral view and occipito-submentovertex view. Waters’ view tends to show all facial structures the best. Lines of Dolan (I–III) are three anatomic contours that correspond to facially important structures.

36

You see a patient in clinic with a malignant thyroid swelling.Which one of the following is correct regarding the types of thyroid cancer?

Anaplastic cancer has a good prognosis

Anaplastic cancer is commoner in younger patients

Follicular carcinoma cannot be diagnosed on fine needle aspiration (FNA) « YOUR ANSWER

Follicular carcinoma is the most common type

Medullary cancer only occurs in patients with multiple endocrine neoplasia type II (MEN II)

YOUR ANSWER WAS CORRECT The Answer Comment on this Question

Anaplastic cancer has a very poor prognosis, and is usually seen in older patients. Hypoparathyroidism is a well recognised complication of surgery. Follicular carcinoma cannot be differentiated from follicular adenoma by FNA. Medullary cancer of the thyroid is associated with the MEN II syndrome but most patients who have medullary cancer will not have MEN. Female to male ratio for thyroid cancer is 3 : 1. Papillary adenocarcinoma is the commonest type (70%).

Page 29: 1filedownload.com · Web view2014/12/19  · Von-Hippel Lindau disease is a rare inherited pathology that causes cyst development in multiple organs, including the pancreas. 52 A

37

You are reviewing a child in the ENT outpatient clinic following a tonsillectomy some weeks ago.The palatine tonsillar bed:

Contains the superior pharyngeal nerve

Is drained by the external palatine vein « YOUR ANSWERIs floored by the middle constrictor muscle

Is pierced by branches of the mandibular artery

Lies in the oral cavity

YOUR ANSWER WAS CORRECT The Answer

The palatine tonsil lies in the oropharynx, in a pit floored by the superior constrictor muscle, through which the IXth cranial nerve passes. The tonsil is supplied by the tonsillar branch of the facial artery. Venous blood first drains into the tonsillar venous plexus, then into the pharyngeal venous plexus and thence into the external palatine vein (a bleeding point after tonsillectomy), or the facial and pharyngeal veins.

38

A 25-year-old male attends hospital on a Friday night, after being involved in a fight. His nose is clinically deformed and bleeding.

Which complication of nasal fractures requires urgent treatment?

Associated facial fracture confirmed with facial view X-rays « YOUR ANSWERDeformity of the nose

Minor associated epistaxis

Nasal septal fracture

Nasal septal haematoma « CORRECT ANSWER

Page 30: 1filedownload.com · Web view2014/12/19  · Von-Hippel Lindau disease is a rare inherited pathology that causes cyst development in multiple organs, including the pancreas. 52 A

YOUR ANSWER WAS INCORRECT The Answer Comment on this Question

A numb cheek suggests damage to infraorbital nerve, most commonly associated with infraorbital rim fracture, not usually nasal fracture. Bony manipulation under anaesthesia (MUA) must be performed within the first 2 weeks of injury, otherwise reduction is difficult and the patient usually needs a formal rhinoplasty. Septal haematoma and subsequent abscess formation is potentially life threatening and so must be excluded at initial examination. X-rays are not helpful in nasal fractures; epistaxis is seldom troublesome.

39

A 23-year-old male is brought by friends into A and E after falling downstairs on leaving a nightclub. On examination, he is slightly drowsy, smelling of alcohol and complaining of neck pain.What should be the immediate management of this patient?

Analgesia

C spine immobilisation « YOUR ANSWERImmediate head and neck CT

IV access and fluid resuscitation

IV Pabrinex

YOUR ANSWER WAS CORRECT The Answer Comment on this Question

The fact a patient may be mobilising on admission to A and E should not distract from the nature of the injury and appropriate initial management. Given the mechanism of the injury and the complaint of neck pain, a traumatic injury to the C-spine is feasible and therefore this case should be dealt with using ATLS principles.

40

Page 31: 1filedownload.com · Web view2014/12/19  · Von-Hippel Lindau disease is a rare inherited pathology that causes cyst development in multiple organs, including the pancreas. 52 A

A 72-year-old farmer presents with a lesion on his left cheek which has been present for several months. On examination, it has nodular appearance with rolled edges and visible telangiectasia. Clinically it is thought to be a basal cell carcinoma (BCC) and the decision is made to proceed with treatment.What proportion of BCCs occur in the head and neck region?

Select one answer only

1%

10%

30%

50%

90% « YOUR ANSWER

YOUR ANSWER WAS CORRECT The Answer Comment on this Question

BCC is the commonest skin tumour on the face and 90% of all BCCs occur in the head and neck region. Sunlight exposure and genetic factors are the main risk factors. BCCs rarely, if ever, metastasise and treatment is with complete surgical excision. Radiotherapy is reserved for recurrences, which are typically aggressive if they recur on the cheeks, nasolabial folds, medial canso and preauricular region.The phrase ‘rather a large scar than a small tomb’ (Sir Harold Gillies) should always be taken into consideration when planning surgical excision – tumours under 1 cm should have at least a 5 mm excision margin and those over 1 cm should have at least a 1 cm margin.

41

A 27-year-old is brought into Accident & Emergency by ambulance following a high speed road traffic accident.What is the best method to immobilise his cervical spine?

With at least a rigid collar in all patients involved in traffic collisions of over 30 miles per hour

Page 32: 1filedownload.com · Web view2014/12/19  · Von-Hippel Lindau disease is a rare inherited pathology that causes cyst development in multiple organs, including the pancreas. 52 A

With a soft collar in all patients with blunt trauma above the clavicles

With a rigid collar, sandbags and tape in all unconscious blunt trauma patients « YOUR ANSWER

In the position that it is found until a fracture has been excluded radiologically

Manually with in-line traction until it has been secured with a collar, sandbags and tape in all unconscious blunt trauma patients

YOUR ANSWER WAS CORRECT The Answer Comment on this Question

The cervical spine must be immobilised in all unconscious victims of trauma, those with blunt injury above the clavicle and those with multisystem trauma. The neck is initially immobilised manually in the in-line position NOT with traction. The head and neck may be carefully moved into the in-line position if found in a different position, however if any resistance is encountered it is then immobilised in the position that it was found. For immobilisation to be adequate a collar, sandbags and tape must all be used.

42

What is the function of the middle nasal meatus?

It contains the bulla ethmoidalis of the middle ethmoidal air sinus « YOUR ANSWER

It drains the nasolacrimal duct

It drains the posterior ethmoidal air sinus

It drains the sphenoidal air cells

It is lined by olfactory epithelium containing the primary olfactory neurones

YOUR ANSWER WAS CORRECT The Answer

The nasolacrimal duct drains into the inferior meatus; the sphenoidal air sinus drains into the sphenoethmoidal recess;

Page 33: 1filedownload.com · Web view2014/12/19  · Von-Hippel Lindau disease is a rare inherited pathology that causes cyst development in multiple organs, including the pancreas. 52 A

and the posterior ethmoidal air sinus drains into the superior meatus. The middle meatus contains the bulla ethmoidalis of the middle ethmoidal air sinus, which drains through a hiatus in the bulla. Olfactory epithelium containing the primary olfactory neurones lines the superior nasal recess (roof of the nasal cavity) under cover of the cribriform plate of the ethmoid bone.

43

A 35-year-old man presents to hospital with progressive deterioration of his vision over a 3-month period. His left visual acuity was 5/20. Enhanced orbital computed tomographic (CT) scans revealed a dilatation of the ophthalmic artery. 

What is the next most appropriate step in his management?

Cerebral angiography of the internal carotid artery to assess the anatomy of the aneurysm « YOUR ANSWER

Immediate surgery to clip the aneurysm to prevent further deterioration in vision

Immediate treatment with high dose corticosteroids is indicated because this presentation is most likely to be secondary to giant cell arteritis

No immediate action in required. This patient should be followed up with serial orbital CT scans every 3 months

Selective cerebral angiography of the external carotid artery to assess the anatomy of the aneurysm

YOUR ANSWER WAS CORRECT The Answer Comment on this Question

The ophthalmic artery is a branch of the internal carotid artery (ICA); it passes through the optic canal and gives off anterior and posterior ethmoidal branches and the supra-orbital branch to the forehead.

Ophthalmic artery aneurysms usually occur on the carotid-ophthalmic junction. True ophthalmic artery aneurysms are rare. Whilst the chance of rupture is low the proximity of such aneurysms to the optic nerve as they pass through the optic canal often causes deterioration in vision. Visual symptoms respond poorly to surgical treatment. In this case the next step is to assess the aneurysm

Page 34: 1filedownload.com · Web view2014/12/19  · Von-Hippel Lindau disease is a rare inherited pathology that causes cyst development in multiple organs, including the pancreas. 52 A

using angiography. The size, shape and location of the aneurysm a decision will determine whether an open or endovascular approach should be used. The rapid deterioration in the patient’s symptoms warrants prompt further investigation.

Giant cell arteritis is an inflammatory disease of the blood vessels, which mainly affects branches of the external carotid artery. It is more common in men (2:1) with a mean age of onset >55 years. People present with; bruits, fever, headache, jaw and tongue claudication, tenderness over the scalp and visual disturbance. Treatment is with high dose corticosteroids.

44

The total length of the trachea is 10cm.At what level does the trachea begin? 

Bifurcation of common carotid artery

Cricoid cartilage « CORRECT ANSWERHyoid bone

Suprasternal notch « YOUR ANSWERThyroid cartilage

YOUR ANSWER WAS INCORRECT The Answer

The trachea commences just below the cricoid cartilage (at the level of C6). Within the thorax and on the right, the trachea is in contact with

the pleura, vagus and subclavian artery.

On its left, the trachea is in contact with : the left recurrent laryngeal nerve, aortic arch the left common carotid artery and subclavian artery.

The trachea ends at the upper border of T5, where it bifurcates.

Page 35: 1filedownload.com · Web view2014/12/19  · Von-Hippel Lindau disease is a rare inherited pathology that causes cyst development in multiple organs, including the pancreas. 52 A

45

Following a left carotid endarterectomy, a 74-year-old man is found to have altered tongue movements. When asked to protrude his tongue, it deviates to the left.Which of the following structures could have been affected intra-operatively to cause this?

Ansa cervicalis

Hypoglossal nerve « YOUR ANSWERLingual nerve

Marginal mandibular nerve

Vagus nerve

YOUR ANSWER WAS CORRECT The Answer Comment on this Question

Damage to the hypoglossal nerve will result in weakness of the extrinsic muscles of the tongue on the ipsilateral side. The tongue will therefore deviate.

46

A 22-year-old female student presents with epistaxis. This fails to improve with pressure.Which of the following methods would be attempted next?

Embolisation 

External carotid artery ligation

Sphenopalatine artery ligation

Trans-sphenoidal ligation

Use of silver nitrate « YOUR ANSWER

YOUR ANSWER WAS CORRECT The Answer

Page 36: 1filedownload.com · Web view2014/12/19  · Von-Hippel Lindau disease is a rare inherited pathology that causes cyst development in multiple organs, including the pancreas. 52 A

Comment on this QuestionSilver nitrate or chemical cautery is often the first method tried and can be very effective. Ligation of local (sphenopalatine) and, in severe life-threatening bleeds, regional (external carotid) arteries are not uncommonly seen. Maxillary artery embolisation is becoming a more frequently used alternative.

47

A 20-year-old male was hit on the side of the head by a cricket ball. He initially lost consciousness for a few seconds but then carried on with the game. Later that afternoon he collapsed. 

What is the most likely diagnosis?

An extra-dural haematoma caused by damage to the anterior branch of the middle meningeal artery« CORRECT ANSWER

A sub-dural haematoma caused by damage to the bridging veins on the surface of the brain « YOUR ANSWER

A sub-dural haematoma caused by damage to the parenchymal surface of the brain

An extra-dural haematoma caused by damage to the posterior branch of the middle meningeal artery

Post-traumatic amnesia

YOUR ANSWER WAS INCORRECT The Answer

Bleeding from the middle meningeal artery following head injury usually leads to an extradural haematoma. This is usually a tear of the anterior branch of the middle meningeal artery, with an underlying linear skull fracture. The characteristic picture is of a head injury with a brief episode of unconsciousness followed by a lucid interval. The patient then develops a progressive hemiparesis, stupor and rapid transtentorial coning with an ipsilateral dilated pupil. Bilateral fixed dilated pupils, tetraplegia and death follow this.

Subdural haematomas do not present in this manner. In a young patient the

Page 37: 1filedownload.com · Web view2014/12/19  · Von-Hippel Lindau disease is a rare inherited pathology that causes cyst development in multiple organs, including the pancreas. 52 A

blood usually arises from the damaged brain surface. In these cases there is underlying brain injury and so no 'lucid' interval.

Post traumatic amnesia refers to a state of confusion that occurs immediately following a traumatic brain injury. The patient is disorientated and unable to remember events that occur after the injury. The duration of PTA has been shown to be a good indicator of overall outcome following TBI.

48

Which of the following extraocular muscles is most important for eye abduction?

Single best answer question – choose ONE true option onlyLateral rectus muscle « YOUR ANSWERInferior rectus muscle

Superior oblique muscle

Inferior oblique muscle

Medial rectus muscle

YOUR ANSWER WAS CORRECT The Answer Comment on this Question

The lateral rectus muscle is an extra-ocular muscle that serves primarily to abduct the eye from the midline. It arises from the lateral part of the tendinous ring within the posterior orbit, and inserts into the lateral sclera of the eye anterior to the equator but posterior to the sclerocorneal junction. The lateral rectus muscle is innervated by the abducens nerve (CN VI).

49

A 67-year-old man is brought to hospital following an assault. He has a deep laceration over his right temporal scalp which is bleeding profusely. 

Which of the following describes the anatomy of scalp most accurately?

Page 38: 1filedownload.com · Web view2014/12/19  · Von-Hippel Lindau disease is a rare inherited pathology that causes cyst development in multiple organs, including the pancreas. 52 A

Contains lymph nodes

Contains the C1 dermatome

Has motor innervation supplied by the facial nerve « CORRECT ANSWERIs supplied exclusively from branches of the external carotid artery « YOUR

ANSWERIs tightly attached to the cranium

YOUR ANSWER WAS INCORRECT The Answer

From superficial to deep, the scalp has five basic layers, denoted by the mnemonic ‘SCALP’: 1/skin; 2/connective tissue; 3/aponeurosis; 4/loose areolar tissue;

4/and periosteum.

The blood supply to the scalp is from branches of both the internal and external carotid arteries. Two such tributaries from the internal carotid are the supraorbital and supratrochlear arteries. There is no C1 dermatome. The aponeurosis of the scalp is separated from the epicranium by loose connective tissue (the plane of cleavage in scalping), facilitating gliding movements. It contains no lymph nodes. Lymphatic drainage is mainly to the submandibular, preauricular, mastoid and occipital nodes. The occipitofrontalis muscle is supplied by the VIIth cranial nerve.

50

A 19-year-old male is rushed to theatre after being stabbed in the neck during an alleged assault. He is bleeding profusely and on arrival has a pulse of 130 and blood pressure of 76/40. In theatre he is given 4 units of blood and surgical exploration shows a laceration to the external carotid artery.At what level does the common carotid artery bifurcate into the external and internal carotid arteries?

Page 39: 1filedownload.com · Web view2014/12/19  · Von-Hippel Lindau disease is a rare inherited pathology that causes cyst development in multiple organs, including the pancreas. 52 A

Select one answer only

Level of the cricoid cartilage

Level of the hyoid bone

Level of the jugular notch

Lower border of the thyroid cartilage

Upper border of the thyroid cartilage « YOUR ANSWER

YOUR ANSWER WAS CORRECT The Answer Comment on this Question

The carotid artery divides into the external and internal carotid branches at the upper border of the thyroid cartilage. The external carotid artery is deep to the hypoglossal nerve, having been crossed by it at the level of the hyoid bone. The carotid sinus is normally found at the division of the common carotid artery or at the commencement of the internal carotid artery.The jugular or suprasternal notch is at T2 vertebral level, and as such is significantly inferior to the bifurcation of the common carotid artery.

51

A 47-year-old male with a history of alcoholic liver disease presents with a six week history of progressively worsening epigastric pain and distension. Computerised tomography (CT) reveals a 4cm fluid collection in the lesser sac.What is the likely diagnosis?

Hepatocellular carcinoma

Liver abscess

Pancreatic pseudocyst « YOUR ANSWERPerforation of the posterior gastric wall

Von-Hippel Lindau disease

YOUR ANSWER WAS CORRECT The Answer Comment on this Question

Page 40: 1filedownload.com · Web view2014/12/19  · Von-Hippel Lindau disease is a rare inherited pathology that causes cyst development in multiple organs, including the pancreas. 52 A

Pancreatic pseudocysts are a complication of both acute and chronic pancreatitis. The incidence of all three is increased in patients with alcoholic liver disease. They usually develop within the lesser sac, and so can be drained via a posterior wall gastrostomy at endoscopy. Von-Hippel Lindau disease is a rare inherited pathology that causes cyst development in multiple organs, including the pancreas.

52

A 69-year-old diabetic lady with a history of a stroke affecting the right cerebral hemisphere is referred to the vascular surgeons as a Carotid duplex of the left shows a 75% stenosis of the internal carotid artery (ICA).Which of the following vessels is the first to arise from the ICA?

Select one answer only

Anterior cerebral artery

Middle cerebral artery

Ophthalmic artery « YOUR ANSWERPosterior cerebral artery

Superior thyroid artery

YOUR ANSWER WAS CORRECT The Answer

The ICA does not give rise to any branches within the neck, the superior thyroid artery is usually the first branch of the external carotid artery.The ophthalmic artery is a branch of the internal carotid and enters the orbit through the optic foramen. The ophthalmic artery then crosses the optic nerve to the medial side of the orbit accompanied by the nasociliary nerve.

The supraorbital and nasal arteries are two branches of the ophthalmic artery that supply part of the skin of the forehead. The ICA bifurcates into the anterior and middle cerebral arteries. The posterior cerebral artery is formed by the bifurcation of the basilar artery which arises from the junction of the left and right vertebral arteries.

Page 41: 1filedownload.com · Web view2014/12/19  · Von-Hippel Lindau disease is a rare inherited pathology that causes cyst development in multiple organs, including the pancreas. 52 A

53

A 24-year-old rugby player was injured in the face during a match. Eye injuries are common in sports

Which complication results from fractures of the orbital floor ('blowout fractures')?

Damage to the mental nerve leading to cheek numbness « YOUR ANSWERDiplopia especially on upward gaze « CORRECT ANSWERHyphema

Ocular muscles rupture

Retinal detachment

YOUR ANSWER WAS INCORRECT The Answer Comment on this Question

Blowout fractures occur along the orbital floor, as this is the thinnest part of the bone. Structures may herniate through into the ethmoidal or maxillary sinuses, causing the ‘trapdoor’ appearance on plain X-ray. Ocular injury occurs in 24% of cases, with enophthalmos and diplopia the commonest signs. Computed tomography (CT) helps to delineate the fracture and aids in planning surgery if required. Damage to the infraorbital nerve leads to cheek numbness.

54

A 12-year-old girl is brought to her GP by her mother with a lump noticed in her neck. It is painless and she is well in herself. On examination, it is inferior to the hyoid bone and in the midline. On protruding her tongue, it is noted to move up.What is the most likely cause of this lump?

Select one answer only

Branchial cyst

Infectious mononucleosis

Metastatic carcinoma

Page 42: 1filedownload.com · Web view2014/12/19  · Von-Hippel Lindau disease is a rare inherited pathology that causes cyst development in multiple organs, including the pancreas. 52 A

Pharyngeal pouch

Thyroglossal cyst « YOUR ANSWER

YOUR ANSWER WAS CORRECT The Answer Comment on this Question

Thyroglossal cysts are found in the midline and rise up on protruding the tongue. They originate due to the thyroid beginning in the foramen caecum at the back of the tongue in embryology and descending to its final position.

Branchial cysts arise due to the non-disappearance of the cervical sinus (where the 2ndbranchial arch grows down over the 3rd and 4th). They are mostly found under the anterior border of sternocleidomastoid where the upper third meets the middle third.

Pharyngeal pouches may protrude into the posterior triangle on swallowing. These occur mostly on the left and a pharyngeal pouch is a pulsion diverticulum that protrudes between the fibres of the inferior pharyngeal constrictor.

55

A 50-year-old patient with a malignant submandibular tumour is undergoing a submandibular gland excision with which you are assisting. 

Which of these structures lies lateral to the superficial part of the submandibular gland?

Cervical branch of the facial nerve

Deep lingual vein

Facial artery « YOUR ANSWERLingual nerve

Mylohyoid

YOUR ANSWER WAS CORRECT The Answer Comment on this Question

Page 43: 1filedownload.com · Web view2014/12/19  · Von-Hippel Lindau disease is a rare inherited pathology that causes cyst development in multiple organs, including the pancreas. 52 A

The facial artery, medial pterygoid and submandibular fossa of the mandible lie lateral to the superficial part of the gland. The facial vein and cervical branch of the facial nerve lie inferior. Mylohyoid, hyoglossus, the lingual nerve, the submandibular ganglion, the hypoglossal nerve, and the deep lingual vein lie medially.

56

A 42-year-old male presents with chronic swelling of his jaw. An OPG shows a jaw cyst in the body of his left mandible.What would be the preferred treatment option?

Enucleation « CORRECT ANSWERMarginal mandibulatory

Marsupialisation

Segmental mandibulectomy « YOUR ANSWERWide local excision

YOUR ANSWER WAS INCORRECT The Answer Comment on this Question

Jaw cysts are the most common cause of chronic swelling of the jaw. They grow slowly, displacing rather than absorbing teeth. They are usually symptom-less unless infected, and are rarely large enough to cause pathological fractures. Pseudocysts do not contain epithelial linings, but are still classified as cysts.

The cysts are benign and therefore wide local excision or removal of significant amounts of surrounding bone are not required. Both enucleation and marsupialisation are valid treatment options and can be combined, with marsupialisation being more commonly used if infection is present to allow this to drain, whereas enucleation removes the cyst itself.

57

A 68-year-old male is seen in a routine outpatient follow up 6 weeks after a right

Page 44: 1filedownload.com · Web view2014/12/19  · Von-Hippel Lindau disease is a rare inherited pathology that causes cyst development in multiple organs, including the pancreas. 52 A

submandibular gland excision. Movement of his face, neck and tongue are all normal, but he is noted to have ulceration on the right lateral border of the tongue.Injury to which of the following structures could have caused this?

Hypoglossal nerve

Inferior alveolar nerve

Lingual nerve « YOUR ANSWERMarginal mandibular nerve

Spinal accessory nerve

YOUR ANSWER WAS CORRECT The Answer Comment on this Question

Complications of submandibular gland excision are haematoma formation, marginal mandibular nerve palsy, lingual nerve and hypoglossal nerve damage.

The lingual nerve carries sensory fibres and taste fibres to the anterior two-thirds of the tongue. Loss of sensation would give rise to increased tendency for tongue biting and hence ulceration. Hypoglossal nerve palsy manifests itself in weakness of the extrinsic muscles of the tongue on the ipsilateral side. Because of the presence of other major and minor salivary glands a dry mouth is not a commonly encountered problem. Frey’s syndrome is a complication of parotidectomy. The accessory nerve is not encountered in this procedure.

58

Following parotid gland surgery in a 51-year-old female, the facial nerve is damaged.

What is the most likely consequence for the patient?

Altered taste to the anterior 2/3 of the tongue

Page 45: 1filedownload.com · Web view2014/12/19  · Von-Hippel Lindau disease is a rare inherited pathology that causes cyst development in multiple organs, including the pancreas. 52 A

Difficulty in chewing food therefore weakness in the muscle of mastication

Inability to rotate the neck because of weakness in sternocleidomastoid

Slurred speech due to weakness in tongue movements

Weakness in buccinator and orbicularis oris « YOUR ANSWER

YOUR ANSWER WAS CORRECT The Answer

As part of its course, having exited the stylomastoid foramen and passed between the mastoid process and tympanic ring, the facial nerve comes to lie between the deep and superficial portions of the parotid gland. Therefore it is at risk of injury during parotidectomy. Within the gland the facial nerve divides into its superior and inferior divisions. The branches given off are the temporal, zygomatic, buccal, mandibular and cervical branches. The buccal branch provides the nerve supply to buccinators and orbicularis oris.

Taste to the anterior two thirds of the tongue is supplied by the chorda tympani, this branch of the facial nerve is given off prior to the nerve entering the parotid gland. The muscles of mastication are predominantly supplied by the trigeminal nerve, (masseter, temporalis and the pterygoids). Sternocleidomastoid is suppled by the accessory nerve. The muscles which move the tongue are: palatoglossus, which is supplied by the pharyngeal plexus and accessory nerve, and genioglossus, hyoglossus and styloglossus, which are supplied by the hypoglossal nerve.

59

An 82-year-old woman fainted and fell downstairs sustaining head injuries with skull fracture. Bleeding was discovered from the middle meningeal artery.What is this is associated with?

Injury to the posterior branch of maxillary artery

Extradural haematoma « YOUR ANSWERShifting of the brain causing ipsilateral pupillary constriction

Page 46: 1filedownload.com · Web view2014/12/19  · Von-Hippel Lindau disease is a rare inherited pathology that causes cyst development in multiple organs, including the pancreas. 52 A

Basal skull fracture

Decrease in the intracranial pressure

YOUR ANSWER WAS CORRECT The Answer Comment on this Question

The anterior branch is more often affected than the posterior, probably because it courses through thinner bone and across the sutures at the pterion. Although designated an extradural bleed, the haematoma collects between the endosteal and meningeal layers of the dura mater. Fractures invariably precede an extradural bleed. The pia mater invests the blood vessel, not the arachnoid mater.

60

Whilst taking the anatomy viva section of the MRCS examination the examiner takes you to a neck prosection. She then points to a tubular structure lying anterior to scalenus anterior, which receives a branch from the retromandibular vein before piercing the deep cervical fascia and joining the subclavian vein. 

What is this structure?

An anatomical variation, as no such structure has this anatomy

The common carotid artery

The external carotid artery

The external jugular vein « YOUR ANSWERThe internal jugular vein

YOUR ANSWER WAS CORRECT The Answer Comment on this Question

The external jugular vein drains most of the scalp and side of the face. It begins near the angle of the mandible and is formed from the union of retromandibular and postauricular veins, receiving branches from the posterior external and transverse cervical veins.

Page 47: 1filedownload.com · Web view2014/12/19  · Von-Hippel Lindau disease is a rare inherited pathology that causes cyst development in multiple organs, including the pancreas. 52 A

The external jugular vein has two pairs of valves, which do not prevent regurgitation of the blood, or the passage of injection from below upward. The lower pair are placed at its entrance to the subclavian vein, the upper (in most cases) about 4cm above the clavicle. The external jugular vein lies anterior to scalenus anterior and pierces the deep fascia of the neck, usually posterior to the clavicular head of the sternocleidomastoid muscle before draining into the subclavian vein.

61

A 70-year-old lady presents with bilateral buttock pain which worsens with prolonged standing and improves with sitting, and also neurological claudication.

Which structure of the lumbar spinal canal is responsible for this spinal stenosis?

Normal facet joints

Normal intervertebral disc anteriorly

Synovial facet cysts posteriorly « YOUR ANSWERThe ligamentum flavum posteriorly « CORRECT ANSWERThe posterior longitudinal ligament posteriorly

YOUR ANSWER WAS INCORRECT The Answer Comment on this Question

The spinal canal is anterior to the ligamentum flavum but posterior to the vertebral disc. The spinal cord ends at L1.

Page 48: 1filedownload.com · Web view2014/12/19  · Von-Hippel Lindau disease is a rare inherited pathology that causes cyst development in multiple organs, including the pancreas. 52 A

62

The sphenopalatine artery:

Single best answer question – choose ONE true option onlyAccompanies the infraorbital nerve to its termination

Enters the infratemporal fossa through the pterygomaxillary fissure

Supplies blood to the lateral nasal wall and nasal septum « CORRECT ANSWER

Is a terminal branch of the middle superior alveolar artery

Sends a branch into the pharyngeal canal « YOUR ANSWER

YOUR ANSWER WAS INCORRECT The Answer Comment on this Question

The sphenopalatine artery, a branch of the third part of the internal maxillary artery, passes through the sphenopalatine foramen into the cavity of the nose, at the back part of the superior meatus. Here it gives off its posterior lateral nasal branches, which spread forward over the conchae and meatuses, anastomose with the ethmoidal arteries and the nasal branches of the descending palatine and assist in supplying the frontal, maxillary, ethmoidal and sphenoidal sinuses. Crossing the undersurface of the sphenoid, the sphenopalatine artery ends on the nasal septum as the posterior septal branches; these anastomose with the ethmoidal arteries and the septal branch of the superior labial. One branch

Page 49: 1filedownload.com · Web view2014/12/19  · Von-Hippel Lindau disease is a rare inherited pathology that causes cyst development in multiple organs, including the pancreas. 52 A

descends in a groove on the vomer to the incisive canal and anastomoses with the descending palatine artery.

63

A 78-year-old woman presents to A&E complaining of neck pain, without neurology, following a fall from standing. A lateral x-ray of the neck reveals a retropharyngeal haematoma. The doctor in A&E suspects an odontoid peg fracture. 

What is the next most appropriate investigation?

A CT scan of the cervical spine « CORRECT ANSWERA CT scan of the whole spine

A magnetic resonance imaging (MRI) scan of the neck

An ‘open mouth’ view x-ray « YOUR ANSWERNo further imaging is required, a retropharyngeal haematoma is diagnostic of an

odontoid peg fracture

YOUR ANSWER WAS INCORRECT The Answer

The odontoid peg is the ascension of the atlas fused to the ascension of the axis. The peg has an articular facet at its front and forms part of a joint with the anterior arch of the atlas. It is a non-weight-bearing joint. The alar ligaments, together with the apical ligaments, are attached from the sloping upper edge of the odontoid peg to the margins of the foramen magnum. The inner ligaments limit rotation of the head and are very strong. The weak apical ligament lies in front of the upper longitudinal bone of the cruciform ligament, and joins the apex of the deltoid peg to the anterior margin of the foramen magnum. It is the fibrous remnant of the notochord.

Prior to CT scanning an ‘open mouth’ view was used to look for PEG fractures. In current practise a CT scan is the gold standard. MRI scans are used to look for ligamentous injury and should be considered after CT scanning. Given the low energy mechanism of injury and in the absence of other symptoms there is no requirement to image the whole spine and this would expose the patient to

Page 50: 1filedownload.com · Web view2014/12/19  · Von-Hippel Lindau disease is a rare inherited pathology that causes cyst development in multiple organs, including the pancreas. 52 A

unnecessary radiation.

A retropharyngeal is not specific for a PEG fracture and can occur in trauma to the neck.

64

You are in the neurosurgical clinic reviewing a patient who has had a left temporal lobe tumour resected and now has a suspected abscess. He has raised inflammatory markers, fever and headache. 

A dominant hemisphere temporal lobe abscess is most likely to cause which symptom?

Motor disturbance in the contralateral arm

Problems with balance

Problems with speech (dysphasia) « YOUR ANSWERSensory disturbance in the contralateral leg

Visual disturbance

YOUR ANSWER WAS CORRECT The Answer Comment on this Question

Cerebral abscesses act as space-occupying lesions and produce focal neurological deficits. They may also cause meningitis, epilepsy and intracranial herniation. Dysphasia occurs with dominant (most commonly left) temporal lobe lesions. The ‘important’ cortical areas of the temporal lobe area are the auditory cortex and temporal association area (responsible for the recognition of auditory stimuli and integration with other modalities).

65

You are assisting your ENT consultant performing a thyroidectomy for malignancy.

Regarding the surgical anatomy of the thyroid gland:

Page 51: 1filedownload.com · Web view2014/12/19  · Von-Hippel Lindau disease is a rare inherited pathology that causes cyst development in multiple organs, including the pancreas. 52 A

The inferior parathyroid glands are more constant in position than the superior parathyroid glands

The middle thyroid veins are more constant in position than the superior and inferior thyroid veins

The thyroid gland does not have a definite capsule

There is a ligament, Berry’s ligament connecting the thyroid to the cricoid cartilage and upper trachea« YOUR ANSWER

Unilateral recurrent laryngeal nerve division results in the contralateral vocal cord lying in the mid- or cadaveric position

YOUR ANSWER WAS CORRECT The Answer

The thyroid gland has a definite, fine capsule, which allows a capsular dissection to preserve the recurrent laryngeal nerves. The superior parathyroid glands are more constant in position than the inferior. Because of their embryological migration, the inferior glands may be situated among the pretracheal lymph nodes or in the thymus as far as 10 cm from the thyroid. The middle thyroid veins are the least constant of the thyroid veins. The superior veins drain into the internal jugular vein; the inferior veins are very constant and drain into the brachiocephalic veins; and the middle veins are very variable and often multiple. Unilateral recurrent laryngeal nerve section results in the ipsilateral vocal cord lying motionless in the mid- or cadaveric position. The voice is hoarse and weak. If both recurrent laryngeal nerves are divided, then the glottic space is narrowed and stridor develops.

66

A 23-year-old male is brought into A&E as a trauma call after being stabbed in the neck by a rival gang member. The knife has been left in place and is entering through sternocleidomastoid on the left below the cricoid cartilage. His heart rate is 80 and blood pressure 140/90. He is alert and able to speak in sentences.

Page 52: 1filedownload.com · Web view2014/12/19  · Von-Hippel Lindau disease is a rare inherited pathology that causes cyst development in multiple organs, including the pancreas. 52 A

What is your management plan?Barium swallow

CT angiogram of head and neck « CORRECT ANSWEREndoscopy

Immediate transfer to theatre for exploration « YOUR ANSWERRemove knife in A&E and pack wound

YOUR ANSWER WAS INCORRECT The Answer Comment on this Question

The patient in this question has sustained a penetrating neck injury to zone 1, and is haemodynamically stable at present as judged by the observations given. It is not necessary to take the patient immediately to theatre, and imaging the affected area can assess the extent of damage and help plan the nature of intervention required. In zone 1 injuries as the thoracic inlet is involved, the thorax may need to be surgically explored, and therefore cardiothoracic surgeons maybe needed.

The zones of penetrating neck injury are:Zone 1 - Clavicle to cricoid cartilageZone 2 - Cricoid cartilage to angle of mandibleZone 3 - Angle of mandible to base of skull

67

A 7-year-old boy is seen by his GP for otitis media of his left ear, on further questioning it emerges he has had recurrent upper respiratory tract infections and has erythematous swollen tonsils at present.Which part of the skull does the Eustachian tube penetrate?

Select one answer only

Greater wing of the sphenoid

Lesser wing of the sphenoid

Occipital bone

Petrous temporal bone « YOUR ANSWER

Page 53: 1filedownload.com · Web view2014/12/19  · Von-Hippel Lindau disease is a rare inherited pathology that causes cyst development in multiple organs, including the pancreas. 52 A

Squamous temporal bone

YOUR ANSWER WAS CORRECT The Answer Comment on this Question

The Eustachian (Phanyngotympanic, auditory) tube connects the tympanic cavity to the nasopharynx. The posterolateral third is bony and the remainder cartilaginous.The Eustachian tube in a child is shorter and more horizontal. The opening of the auditory tube lies above the soft palate adjacent to the tubal tonsil. The bony part of the Eustachian tube perforates the petrous temporal bone.

68

A 56-year-old heavy smoker is referred urgently to a maxillofacial surgery department with a suspected oral cavity malignancy by his GP.Which of the following is the most common site for an oral cavity malignancy?

Select one answer only

Hard palate

Mucosa of vestibule

Mucosa overlying palatoglossal arch

Soft palate

Tongue « YOUR ANSWER

YOUR ANSWER WAS CORRECT The Answer Comment on this Question

Oral cavity malignancies are usually squamous cell carcinomas and 35% occur on the tongue. They metastasise to cervical lymph nodes commonly and are related to risk factors such as smoking, alcohol, malnutrition, betel nut chewing and dental caries. Surgical excision with radiotherapy is the usual treatment of choice.

69

Page 54: 1filedownload.com · Web view2014/12/19  · Von-Hippel Lindau disease is a rare inherited pathology that causes cyst development in multiple organs, including the pancreas. 52 A

A 45-year-old man presents to clinic with pain and parotid swelling which is worse with eating. You palpate a stone in the parotid duct.The parotid duct:

Has an opening located on the floor of the mouth next to the frenulum

Has an opening opposite the first lower molar

Is approximately 1 cm long

Lies in the middle third of a line between the intertragic notch of the auricle and the mid-point of the philtrum« YOUR ANSWER

Runs between mylohyoid and hyoglossus

YOUR ANSWER WAS CORRECT The Answer

The parotid duct is approximately 5 cm long. It crosses the masseter, turning around its anterior border to pass through the buccal fat pad and pierce the buccinator. When intraoral pressure is raised the submucous part of the parotid duct is compressed by the buccinator. The parotid gland is mainly a serous gland. The parotid duct opens on the middle third of a line between the intertragic notch of the auricle and the midpoint of the philtrum, opposite the second upper molar. The submandibular duct opening is located on the floor of the mouth, next to the frenulum and this duct runs between mylohyoid and hyoglossus.

70

Each branchial (pharyngeal) arch has a cartilaginous bar, a muscle component that differentiates from the cartilaginous tissue, an artery and a cranial nerve. The first pharyngeal (branchial) arch:

Single best answer question – choose ONE true option onlyGives rise to the styloid process and hyoid bone « YOUR ANSWERGives rise to the palatine tonsil

Page 55: 1filedownload.com · Web view2014/12/19  · Von-Hippel Lindau disease is a rare inherited pathology that causes cyst development in multiple organs, including the pancreas. 52 A

Gives rise to the muscles of facial expression

Gives rise to the sphenomandibular ligament « CORRECT ANSWERIs innervated by the glossopharyngeal nerve

YOUR ANSWER WAS INCORRECT The Answer

The first pharyngeal arch or mandibular arch is involved with development of the face. It develops two processes, maxillary and mandibular, which form the upper and lower jaws respectively. Bones and muscles of this region are developed from mesoderm in the arch. Meckel’s cartilage is the first arch cartilage. It ossifies to form the malleus and incus in the middle ear. The sphenomandibular ligament is derived from its perichondrium. The rest of the cartilage disappears after the mandible forms around it by intramembranous ossification. The muscles derived from the first arch include temporalis, masseter, medial and lateral pterygoids, anterior belly of the digastric, mylohyoid, tensor tympani and tensor palati. The trigeminal nerve is the motor supply of the mandibular arch.

71

Whilst performing an elective routine tracheostomy, heavy bleeding occurs intra-operatively.Which of the following vessels is most likely to be the source of this?

Anterior jugular vein « YOUR ANSWERExternal jugular vein

Internal jugular vein

Middle thyroid vein

Superior thyroid vein

YOUR ANSWER WAS CORRECT The Answer

Page 56: 1filedownload.com · Web view2014/12/19  · Von-Hippel Lindau disease is a rare inherited pathology that causes cyst development in multiple organs, including the pancreas. 52 A

Comment on this QuestionInternal and external jugular veins, as well as the middle and superior thyroid veins, are too lateral to be encountered in a routine tracheostomy. Anterior jugular veins and inferior thyroid veins, however, may be encountered.

72

The ophthalmic artery emerges through which of the following foramina to reach the eye?

Single best answer question – choose ONE true option onlyOptic canal « YOUR ANSWERForamen spinosum

Superior orbital fissure

Foramen rotundum

Inferior orbital fissure

YOUR ANSWER WAS CORRECT The Answer Comment on this Question

The ophthalmic artery arises from the internal carotid, just as that vessel is emerging from the cavernous sinus, on the medial side of the anterior clinoid process and enters the orbital cavity through the optic foramen (canal), below and lateral to the optic nerve. It then passes over the nerve to reach the medial wall of the orbit and thence horizontally forward, beneath the lower border of the superior oblique and divides it into two terminal branches, the frontal and dorsal nasal. As the artery crosses the optic nerve it is accompanied by the nasociliary nerve and is separated from the frontal nerve by the rectus superior and levator palpebrae superioris.

73

A patient undergoes a radical parotidectomy for a malignant parotid tumour, at which time it is found necessary to perform a total division of the left facial (VII) nerve at this level. Postoperatively, which is the most likely sequel?

Page 57: 1filedownload.com · Web view2014/12/19  · Von-Hippel Lindau disease is a rare inherited pathology that causes cyst development in multiple organs, including the pancreas. 52 A

Single best answer question – choose ONE true option only

Loss of left sided frown in all cases « YOUR ANSWERNumbness over the cheek on the left side

Ptosis of the upper eyelid on the left side

Loss of taste sensation over the anterior two-thirds of the tongue on the left side

Tendency for food and fluids to collect in the buccal sulcus after meals « CORRECT ANSWER

YOUR ANSWER WAS INCORRECT The Answer Comment on this Question

The facial nerve supplies all the muscles needed for facial expression including the occipitofrontalis, which wrinkles the forehead. A distressing feature is paralysis of the buccinator muscle, which acts to empty the buccal sulcus during mastication. There are no cutaneous sensory fibres in the facial nerve. The levator palpebrae superioris is supplied by the oculomotor nerve, so the patient can still raise his upper lid. The chorda tympani fibres, which transmit taste from the anterior two-thirds of the tongue, pass from the lingual nerve to the facial nerve just below the skull, and therefore remain intact in peripheral injuries of the facial nerve.

74

You are in neurosurgical outpatients’ clinic reviewing a patient with suspected trigeminal neuralgia.The trigeminal nerve:

Has a mandibular division which is purely sensory

Has ophthalmic and maxillary divisions, which are purely motor

Supplies sensation to the angle of the mandible

Supplies the buccinator muscle

Supplies the masseter muscle « YOUR ANSWER

YOUR ANSWER WAS CORRECT The Answer

Page 58: 1filedownload.com · Web view2014/12/19  · Von-Hippel Lindau disease is a rare inherited pathology that causes cyst development in multiple organs, including the pancreas. 52 A

Comment on this QuestionThe trigeminal (V) nerve has sensory fibres to the greater part of the skin of the face, mucous membranes of the mouth, nose and paranasal air sinuses. It provides motor innervation (via its motor, mandibular branch) to the muscles of mastication (temporalis, masseter, pterygoid). The buccinator muscle is supplied by the facial nerve. The angle of the mandible is supplied by the great auricular nerve (C2–C3).

75

A 57-year-old male consults his Dentist with a persistent, non-healthy ulcer at the base of his tongue which bleeds sporadically. The Dentist is concerned this could represent a squamous cell carcinoma and refers him for urgent investigation.Which of the following are risk factors for oral squamous cell carcinoma?

Select one answer only

Diabetes mellitus

HIV « CORRECT ANSWERHypertension

Hypothyroidism

Steroid therapy « YOUR ANSWER

YOUR ANSWER WAS INCORRECT The Answer Comment on this Question

Smoking and high alcohol intake are synergistic risk factors for oral squamous-cell carcinoma (SCC), increasing the risk by six-fold. Other factors include syphilis, malnutrition, cirrhosis, Plummer–Vinson syndrome and leucoplakia. Eighty per cent of acquired immunodeficiency syndrome (AIDS) sufferers develop hairy oral leucoplakia.

76

A 61-year-old female is commenced on anti-hypertensives by her GP as her

Page 59: 1filedownload.com · Web view2014/12/19  · Von-Hippel Lindau disease is a rare inherited pathology that causes cyst development in multiple organs, including the pancreas. 52 A

blood pressure has been elevated on the last three visits. Following her first dose at home, she noted her tongue became very swollen and she had some difficulty breathing. She was rushed to the emergency department and a diagnosis of angio-neurotic oedema was made.Which group of the following anti-hypertensives is most likely to have precipitated this?

Select one answer only

ACE inhibitors « YOUR ANSWERα blockers

β blockers

Calcium channel blockers

Thiazide diuretics

YOUR ANSWER WAS CORRECT The Answer Comment on this Question

Angio-neurotic oedema is tongue swelling secondary to ACE inhibitors. The most important initial management is to secure the airway; a nasotracheal tube may be required. Steroids (which may take 6 hours to take full effect) and antihistamine should be commenced immediately.

77

A 20-year-old girl presented with a unilateral conductive hearing loss; otoscopy revealed characteristics of Cholesteatoma of the middle ear.What would be the most appropriate management plan?

As it is a benign tumour composed of cholesterol it should be left alone

It should be completely removed at initial surgery « YOUR ANSWERIt could be left as it does not pose a risk of meningitis

As it usually presents with a neck abscess it should be treated with incision and drainage

Radiotherapy is appropriate and effective as it is composed of atypical cells

YOUR ANSWER WAS CORRECT

Page 60: 1filedownload.com · Web view2014/12/19  · Von-Hippel Lindau disease is a rare inherited pathology that causes cyst development in multiple organs, including the pancreas. 52 A

The Answer Comment on this Question

Cholesteatoma derived its name from early observations that it resembled a ball of cholesterol. Although given the suffix –oma, it does not behave like a tumour, nor is it composed of atypical cells suggestive of a tumour. It is in fact a non-cleaning squamous cell cyst that can cause complications if untreated, due to its invasive properties, including meningitis, brain abscess, hearing loss, neck abscess (Bezold’s abscess, very rare), lateral sinus thrombosis, facial nerve palsy and vertigo. It usually presents with continuously discharging ears and otalgia. Treatment is regular cleaning and exteriorisation in the form of mastoidectomy and variations of this procedure.

78

A 16-year-girl is brought in by her anxious parents as they have found a lump in her neck. The lump is not painful and not causing any symptoms, on examination you note it is in the midline and moves up when the child protrudes her tongue.Which of the following is the most likely cause for this swelling?

Branchial cyst

Dermoid cyst

Lymphoma

Sebaceous cyst

Thyroglossal cyst « YOUR ANSWER

YOUR ANSWER WAS CORRECT The Answer Comment on this Question

These occur if remnants of the thyroglossal duct persist. As this duct is a midline structure, they occur in the midline and are more common in women mostly presenting between the ages of 15 and 30 years. On examination, they are 2-3cm in diameter, smooth and round and move up on protrusion of the tongue. 

79

Page 61: 1filedownload.com · Web view2014/12/19  · Von-Hippel Lindau disease is a rare inherited pathology that causes cyst development in multiple organs, including the pancreas. 52 A

You are reviewing a patient in the ENT clinic with a painful swelling around the jaw and you suspect there is a mass in the submandibular salivary gland.The submandibular gland:

Has the facial artery running through it « CORRECT ANSWERHas the hypoglossal nerve running through it

Is deep to the hyoglossus muscle « YOUR ANSWERLies entirely below the lower mandible

Lies below the digastric muscle

YOUR ANSWER WAS INCORRECT The Answer Comment on this Question

The submandibular gland consists of a deep and a superficial part. The superficial part lies in the digastric triangle (above and between the two bellies of the digastric muscle). The hypoglossal nerve runs medial to the superficial part of the gland. The gland is superficial to the mylohyoid and hyoglossus muscles. A third of the submandibular gland lies below the lower border of the mandible and two-thirds above it.

80

Which of the following structures is the temporomandibular joint (TMJ) attached to superiorly?

Articular disc

Articular eminence

Articular tubercle « YOUR ANSWERPostglenoid tubercle

Squamo-tympanic fissure « CORRECT ANSWER

YOUR ANSWER WAS INCORRECT The Answer Comment on this Question

Page 62: 1filedownload.com · Web view2014/12/19  · Von-Hippel Lindau disease is a rare inherited pathology that causes cyst development in multiple organs, including the pancreas. 52 A

The temporomandibular joint is a synovial joint situated between the condyle of the mandible below and the mandibular fossa above. Although it a synovial joint, it is lined by fibrous cartilage (rather than hyaline cartilage typical of synovial joints). The joint is surrounded by a capsule that is attached beyond the limits of the articular surfaces. The capsule is strengthened medially and laterally by collateral ligaments. Superiorly, the capsule is attached to the anterior edge of the squamo-tympanic fissure. Medially, it runs along the suture between the temporal and sphenoid bones and attaches anteriorly to the anterior end of the articular eminence. Laterally, the capsule is attached to the articular tubercle, which forms the lateral limit of the articular eminence and the prominent ridge of the bone forming the lateral lip of the glenoid cavity. The non-articular surfaces enclosed within the capsule are lined with synovial membrane. The joint is stable anteriorly but is lax posteriorly as it is attached well below the articular surface to the neck of the condyle.

81

A 62-year-old heavy smoker is noted by his dentist to have a white patch on his oral mucosa during routine assessment. This cannot be scraped off easily.What is the most likely cause for this lesion?

Candidiasis

Erythroplasia

Leucoplakia « YOUR ANSWERLichen planus

Tertiary syphilis

YOUR ANSWER WAS CORRECT The Answer Comment on this Question

All of the above lesions are pre-malignant. High-risk lesions are those that in 25% of cases will become malignant in 5 years. The lesions in leukoplakia cannot be scraped off. High-risk lesions are rare (especially syphilis) but are important to bear in mind when patients present with lesions in the mouth.

82

Page 63: 1filedownload.com · Web view2014/12/19  · Von-Hippel Lindau disease is a rare inherited pathology that causes cyst development in multiple organs, including the pancreas. 52 A

A 25-year-old motorcyclist sustains a head injury and presents with a Glasgow Coma Score of 10. He has no other complications. 

What should the initial investigations of this patient include?Trauma series X-rays of chest, pelvis and cervical spine « YOUR ANSWERInsertion of an intracranial pressure monitor

A lumbar puncture

Arterial blood gases

Electrocardiography (ECG)

YOUR ANSWER WAS CORRECT The Answer Comment on this Question

Plain radiographs of the chest, pelvis and cervical spine should be performed in all patients with a significant head injury who are unconscious and so cannot be assessed reliably. The insertion of an intracranial pressure (ICP) monitor is indicated for diffuse axonal injuries where patients are managed conservatively, persistent raised ICP and in the presence of systemic complications, such as severe hypotension and hypoxia. Lumbar puncture is contraindicated in patients with raised ICP due to the risk of coning.

83

A 58-year-old manual labourer presents with a lesion on his lower lip, he describes a ‘sore’ which bleeds intermittently and isn’t healing. On examination he has a small ulcerated lesion on his lower lip approximately 1x1cm, and he is noted to have palpable lymphadenopathy of the jugulodigastric nodes.What is the most likely cause of this lesion?

Basal cell carcinoma « YOUR ANSWERKeratoacantoma

Leukoplakia

Melanoma

Squamous cell carcinoma « CORRECT ANSWER

Page 64: 1filedownload.com · Web view2014/12/19  · Von-Hippel Lindau disease is a rare inherited pathology that causes cyst development in multiple organs, including the pancreas. 52 A

YOUR ANSWER WAS INCORRECT The Answer Comment on this Question

SCC is the most common malignant oral cavity cancer. Cervical nodal involvement is common at presentation, but usually involves anterior triangle nodes first, before spreading to posterior triangle nodes. Risk factors include smoking, betel nut chewing, alcohol abuse and leucoplakia.

84

A 25-year-old male who is in Burns intensive care having sustained 42% burns in a house fire is on day 12 post injury and remains intubated and ventilated. The Consultant Anaesthetist feels he will not be ready for extubation for a while and the decision is made to perform a tracheostomy. A size 7 tracheostomy tube is used.Which of the following does the tube size relate to?

Select one answer only

Distance from tracheostomy tube tip to carina

Distance of tracheostomy tube from vocal cords

Internal diameter of the tracheostomy tube « YOUR ANSWERLength of the tracheostomy tube

Outer diameter of the tracheostomy tube

YOUR ANSWER WAS CORRECT The Answer Comment on this Question

The first tracheal ring is complete in children and must not be excised for fear of tracheal stenosis. The preferred method is excision of rings 2–4. Tracheostomy should be performed if prolonged intubation is expected; either percutaneous or open approaches are used. Fenestrated tubes allow air to be diverted superiorly through the vocal cords so speech can be possible, but this can also be attained if the tube is occluded with a finger. A size 7 tracheostomy tube relates to an internal diameter (ID) of 7 mm, not the outer diameter or length.

Page 65: 1filedownload.com · Web view2014/12/19  · Von-Hippel Lindau disease is a rare inherited pathology that causes cyst development in multiple organs, including the pancreas. 52 A

85

A 21-year-old boxer presents with a swollen, deformed nose after being knocked out in a fight. He is concerned it is broken and wants you to do something about it.What is the most appropriate management plan?

Attempt early reductions within 48 hours

Consider reduction after 5 days « CORRECT ANSWERCT « YOUR ANSWERFrontal x-ray

Lateral x-ray

YOUR ANSWER WAS INCORRECT The Answer Comment on this Question

The diagnosis of nasal fractures is a clinical one, and X-rays are not usually required. Reduction is required if there is significant cosmetic or functional abnormality, and this is usually carried out 5 days after injury to allow for swelling to subside. Nasal septal haematoma leads to cartilage necrosis if bilateral, and causes saddle-nose deformity..

86

How could you best describe the recurrent laryngeal nerve?

The recurrent laryngeal nerve supplies all of the intrinsic laryngeal muscles

The recurrent laryngeal nerve supplies the cricothyroid muscles

The recurrent laryngeal nerves are sensory to the subglottic region and supply all of the intrinsic muscles except the cricothyroid muscle« YOUR ANSWER

The recurrent laryngeal nerves are sensory to the supraglottic region and supplies the sternohyoid muscle

Page 66: 1filedownload.com · Web view2014/12/19  · Von-Hippel Lindau disease is a rare inherited pathology that causes cyst development in multiple organs, including the pancreas. 52 A

The recurrent laryngeal nerves are sensory to the supraglottic region and supply all of the intrinsic muscles except the cricothyroid muscle

YOUR ANSWER WAS CORRECT The Answer Comment on this Question

The recurrent laryngeal nerves are sensory to the subglottic region and supply all of the intrinsic muscles except the cricothyroid muscle.

87

You are performing a lymph node biopsy in the posterior triangle of the neck and you need to mark out the boundaries of the posterior triangle.These are?

The anterior border of sternocleidomastoid, the mandible and the midline

The anterior border of sternocleidomastoid, the posterior border of trapezius and the clavicle

The anterior border of sternocleidomastoid, the posterior border of trapezius and the scapula

The posterior border of sternocleidomastoid, the anterior border of trapezius and the scapula « YOUR ANSWER

The posterior border of sternocleidomastoid, anterior border of trapezius and the clavicle « CORRECT ANSWER

YOUR ANSWER WAS INCORRECT The Answer Comment on this Question

The posterior triangle lies between the posterior border of sternocleidomastoid, the anterior border of trapezius and the clavicle. The anterior triangle lies between the anterior border of sternocleidomastoid, the mandible and the midline. Lymph node excision from the posterior triangle can result in damage to the accessory nerve. The anterior triangle is subdivided into four further triangles. The common carotid artery bifurcates in the anterior triangle.

Page 67: 1filedownload.com · Web view2014/12/19  · Von-Hippel Lindau disease is a rare inherited pathology that causes cyst development in multiple organs, including the pancreas. 52 A

88

You are treating a patient who is going to undergo parotidectomy and he asks you about parotid neoplasms.Which of the following is correct?

Computed tomography (CT) rather than magnetic resonance imaging (MRI) is the radiological investigation of choice for staging of parotid carcinomas

Presentation with facial nerve palsy implies malignancy « YOUR ANSWERThey are commonly bilateral

They are malignant in 50%

They usually present with maxillary numbness

YOUR ANSWER WAS CORRECT The Answer Comment on this Question

Parotid neoplasms are usually unilateral, only 15–20% of parotid tumours are malignant. The main mode of presentation is a symptomless swelling, often dating back several years. Presentation with facial nerve palsy is very suggestive of a malignancy. Rare presentations include trismus and referred pain via the trigeminal nerve. MRI is used for radiological staging as there is better soft tissue discrimination, imaging can be carried out in multiple planes and it is easier to detect cervical lymphadenopathy.

89

The vocal ligaments:

Single best answer question – choose ONE true option onlyAre formed by the superior free edge of the conus elasticus « CORRECT

ANSWERAre formed by the junction of the aryepiglottic and thyroepiglottic ligaments

Are formed by the inferior free edge of the thyrohyoid ligament

Contain the cricothyroid muscle « YOUR ANSWER

Page 68: 1filedownload.com · Web view2014/12/19  · Von-Hippel Lindau disease is a rare inherited pathology that causes cyst development in multiple organs, including the pancreas. 52 A

Are attached to the superior margin of the epiglottis

YOUR ANSWER WAS INCORRECT The Answer Comment on this Question

The vocal folds are concerned with the production of sound and enclose two strong bands, the vocal ligaments. Each ligament consists of a band of yellow elastic tissue formed by the superior free edge of the conus elasticus, attached in front to the angle of the thyroid cartilage and behind to the vocal process of the arytenoid. Its lower border is continuous with the thin lateral part of the conus elasticus. Its upper border forms the lower boundary of the ventricle of the larynx. Laterally, the vocalis muscle lies parallel with it. It is covered medially by mucous membrane, which is extremely thin and closely adherent to its surface.

90

During thyroidectomy, the inferior laryngeal branch of the right recurrent laryngeal nerve was injured.The action of which of the following laryngeal muscles is most likely to be affected?

Single best answer question – choose ONE true option onlyThyroarytenoid « YOUR ANSWERArytenoid

Lateral cricoarytenoid

Posterior cricoarytenoid « CORRECT ANSWERCricothyroid

YOUR ANSWER WAS INCORRECT The Answer

The posterior cricoarytenoid is innervated by the inferior laryngeal nerve, which is a continuation of the recurrent laryngeal nerve. The posterior cricoarytenoid is the only muscle that abducts the vocal folds. If this muscle is denervated, the

Page 69: 1filedownload.com · Web view2014/12/19  · Von-Hippel Lindau disease is a rare inherited pathology that causes cyst development in multiple organs, including the pancreas. 52 A

vocal folds may be paralysed in an adducted position, which would prevent air from entering the trachea. Arytenoid, lateral cricoarytenoid and thyroarytenoid all adduct the vocal folds. Cricothyroid is the only laryngeal muscle innervated by the external branch of the superior laryngeal. It tenses the vocal ligaments by tipping the thyroid cartilage forward relative to the cricoid cartilage.

91

A 27-year-old man is brought into A&E with facial fractures following a road traffic accident. 

Which step is important in the initial management of facial injuries?

Antibiotics and tetanus immunisation are always required

Early surgical correction is preferable

Facial X-rays are helpful and easy to interpret

Nasal intubation is always indicated

First aid and nasal cautery should be used initially to control nasal bleeding « YOUR ANSWER

YOUR ANSWER WAS CORRECT The Answer Comment on this Question

When dealing with severe facial trauma, advanced trauma life support (ATLS) principles must be adhered to. Therefore in the initial management using the achievable benchmarks of care (ABCs) scenarios, bleeding must be sought and stopped, especially as this can lead to airway compromise. Surgery can usually wait until the patient is fully resuscitated. Chest X-ray must be performed as part of routine imaging, especially if the possibility of inhaled teeth is to be considered. IV antibiotics may be necessary if there is the possibility of open fractures, but are not always necessary.

92

A 25-year-old male is brought to A&E after being assaulted outside a nightclub.

Page 70: 1filedownload.com · Web view2014/12/19  · Von-Hippel Lindau disease is a rare inherited pathology that causes cyst development in multiple organs, including the pancreas. 52 A

On examination there is considerable swelling present, and he is noted to have a reduced sensation in the distribution of his left infraorbital nerve. A CT reveals a fracture through the nasal bones on the ascending process of the maxilla, lacrimal bone and extending into the orbital rim. The orbital floor is otherwise intact.What is the likely injury?

Base of skull fracture

Le fort I fracture

Le fort II fracture « YOUR ANSWERLe fort III fracture

Tripod fracture

YOUR ANSWER WAS CORRECT The Answer Comment on this Question

Fractures to the temporal bone are particularly associated with extradural haematoma. Remember the latent period with these injuries! Le fort III injuries describe a complex fracture which involves the orbital floor, zygomatic arch and maxillary antrum. The mid portion of the face is basically mobile and thus UNSTABLE. Fractures to the paper-thin medial wall of the orbit (lamina papyracea) open the orbit and thus the cranium to the ethmoid air cells (sinus). A ‘tripod’ fracture often involves the orbital floor as well as the lateral wall of the maxillary sinus, and a fluid level in the sinus represents blood and often directs you to the area concerned.

93

A 21-year-old male is seen in A&E after an assault outside a night club with pain in his jaw. An X-ray confirms a fracture of the mandible.Which of the following regions is the weakest part of the mandible?

Select one answer only

Angle

Page 71: 1filedownload.com · Web view2014/12/19  · Von-Hippel Lindau disease is a rare inherited pathology that causes cyst development in multiple organs, including the pancreas. 52 A

Canine region « CORRECT ANSWERCoronoid process

Head

Ramus « YOUR ANSWER

YOUR ANSWER WAS INCORRECT The Answer Comment on this Question

Mandibular fractures are commonly the result of blows to the face and it is not uncommon for both sides to be affected. Fractures of the condyles lead to temporo-mandibular joint dislocation and usually are as a result of a blow to the chin. The canine region is the weakest part of the bone due to the length of the root. Angle fractures can run downwards and forwards, which leads to impaction and so prevents displacement. Fractures of the body always cause laceration to the buccal mucosa.

94

You are reviewing a patient in clinic who has been referred by their GP with a swelling in the neck and episodes of blackouts.Your suspected diagnosis should be:

Carotid body tumour (chemodectoma) « YOUR ANSWERCervical rib

Laryngocele

Pharyngeal pouch

Thyroglossal cyst

YOUR ANSWER WAS CORRECT The Answer

Lateral swellings of the neck include lymph nodes, salivary glands, branchial cysts, cervical ribs, carotid body tumours(chemodectomas), cystic hygromas, sternocleidomastoid tumours, arteriovenous fistulae and lateral thyroid lobe

Page 72: 1filedownload.com · Web view2014/12/19  · Von-Hippel Lindau disease is a rare inherited pathology that causes cyst development in multiple organs, including the pancreas. 52 A

lesions. A chemodectoma is a tumour of the glomus jugulare, and contains chemoreceptor tissue; it may cause attacks of vertigo and syncope.

95

A patient undergoes excision of the left submandibular salivary gland for sialectasia. Unfortunately, his hypoglossal (XII) nerve on that side is damaged. What is the most likely outcome? Single best answer question – choose ONE true option only  

There is numbness of the posterior one-third of the tongue

On protruding the tongue, it deviates towards the right

The uvula deviates towards the left

All the intrinsic muscles of the left side of the tongue are paralysed « YOUR ANSWER

The genioglossus muscle is spared

YOUR ANSWER WAS CORRECT The Answer Comment on this Question

The hypoglossal nerve supplies all the muscles of the tongue but none of the palate (the palatoglossus muscle, supplied by the vagus nerve, is a muscle of the palate). It has no sensory component. The genioglossus muscle protrudes the tongue; when it is paralysed, the muscle on the opposite side is unaffected and deviates the tongue towards the affected side.

96

The submandibular duct opens:

Single best answer question – choose ONE true option onlyNear the maxillary second molar

Page 73: 1filedownload.com · Web view2014/12/19  · Von-Hippel Lindau disease is a rare inherited pathology that causes cyst development in multiple organs, including the pancreas. 52 A

Near the mandibular first molar

From the incisive foramen

Near the midline in the anterior aspect of the floor of the mouth « YOUR ANSWER

Into the buccal vestibule near the mandibular ramus

YOUR ANSWER WAS CORRECT The Answer Comment on this Question

The submandibular duct (Wharton’s duct) is about 5 cm long and its wall is much thinner than that of the parotid duct. It begins from numerous branches from the deep surface of the gland and runs forward between the mylohyoid and the hyoglossus and genioglossus, then between the sublingual gland and the genioglossus and opens by a narrow orifice on the summit of a small papilla, at the side of the frenulum linguae, near the midline in the anterior aspect of the floor of the mouth. On the hyoglossus it lies between the lingual and hypoglossal nerves, but at the anterior border of the muscle it is crossed laterally by the lingual nerve. The terminal branches of the lingual nerve ascend on its medial side.

97

In surgery of the thyroid gland, the external laryngeal nerve may be injured and must be identified before ligating the:

Single best answer question – choose ONE true option onlyInferior thyroid artery

Superior thyroid artery « YOUR ANSWERSuperior pharyngeal artery

Inferior laryngeal artery

Ascending palatine artery

YOUR ANSWER WAS CORRECT The Answer Comment on this Question

Page 74: 1filedownload.com · Web view2014/12/19  · Von-Hippel Lindau disease is a rare inherited pathology that causes cyst development in multiple organs, including the pancreas. 52 A

The external laryngeal nerve is the smaller, external branch (ramus externus) of the superior laryngeal nerve. It descends on the larynx, beneath the sternothyroid muscle, to supply the cricothyroid muscle. It gives branches to the pharyngeal plexus and the superior portion of the inferior pharyngeal constrictor and communicates with the superior cardiac nerve behind the common carotid artery. The external branch is susceptible to damage during thyroidectomy, as it lies immediately deep to the superior thyroid artery.

98

You are inserting a central line into the internal jugular vein.Which of the following is a tributary of the internal jugular vein?

The external jugular vein

The inferior thyroid vein « YOUR ANSWERThe middle thyroid vein « CORRECT ANSWERThe retromandibular vein

The transverse cervical vein

YOUR ANSWER WAS INCORRECT The Answer

The internal jugular vein receives facial, pharyngeal, lingual and superior and middle thyroid veins.

It is formed from the sigmoid and inferior petrosal sinus and continues to the brachiocephalic vein. The external jugular arises from the junction of the posterior auricular vein and the posterior division of the retromandibular vein and drains into the subclavian. The transverse cervical vein is a tributary of the external jugular vein.

99

A 78-year-old male is about to undergo a left Carotid endarterectomy, the

Page 75: 1filedownload.com · Web view2014/12/19  · Von-Hippel Lindau disease is a rare inherited pathology that causes cyst development in multiple organs, including the pancreas. 52 A

anaesthetist has discussed the anaesthetic options with the patient and they have decided to attempt a Cervical plexus block.Which of the following nerves lie superficial to the sternocleidomastoid muscle and may be affected by this block?

Select one answer only

Greater occipital nerve

Hypoglossal nerve

Phrenic nerve

Suprascapular nerve

Traverse cervical nerve « YOUR ANSWER

YOUR ANSWER WAS CORRECT The Answer Comment on this Question

The Cervical plexus is located in the posterior triangle and is formed by union of the anterior rami of the first four cervical nerves. The superficial branches of the cervical plexus are the lesser occipital nerve, greater auricular nerve, transverse cervical nerve and supraclavicular nerve.Local anaesthetic can be injected along the posterior border of the sternocleidomastoid (between the superior and inferior thirds) – the nerve point of the neck for a cervical plexus block. The transverse cervical nerve emerges as a single trunk behind the posterior border of the sternocleidomastoid and is superficial to the muscle.

100

A 54-year-old woman presents with longstanding tinnitus and evidence of a unilateral fifth (trigeminal nerve) palsy. Her MRI scan shows evidence of an acoustic neuroma. Where is compression of the trigeminal nerve most likely to be occurring?Single best answer question – choose ONE true option only 

Cerebellopontine angle « CORRECT ANSWERCavernous sinus

Brainstem

Page 76: 1filedownload.com · Web view2014/12/19  · Von-Hippel Lindau disease is a rare inherited pathology that causes cyst development in multiple organs, including the pancreas. 52 A

Skull base « YOUR ANSWERTrigeminal ganglion

YOUR ANSWER WAS INCORRECT

A complete fifth nerve lesion causes unilateral sensory loss on the face, tongue and buccal mucosa. When motor fibres are damaged there is deviation of the jaw to the side of the lesion as the mouth is opened. Loss of the corneal reflex may be an early indication of a fifth nerve lesion.

Brainstem lesions involving the fifth nerve nuclei may include brainstem glioma, multiple sclerosis, brainstem infarction or syringobulbia.

Lesions at the cerebellopontine angle resulting in fifth nerve damage may include acoustic neuroma, meningioma and secondary tumour deposits.

Within the cavernous sinus, the trigeminal ganglion may be compressed by a pituitary tumour extending into the sinus, internal carotid artery aneurysm, cavernous sinus thrombosis or secondary tumour.

The trigeminal ganglion may also be affected by herpes zoster infection. Prognosis for the recovery of trigeminal nerve function is dependent on the underlying cause.

101

A 76-year-old man underwent tracheostomy for long term ventilation. A few weeks later the tube was removed but the patient had hoarseness of voice.

What is the most likely complication?Surgical emphysema

Pneumonia

Air embolism

Haemorrhage

Subglottic stenosis « YOUR ANSWER

YOUR ANSWER WAS CORRECT The Answer

Page 77: 1filedownload.com · Web view2014/12/19  · Von-Hippel Lindau disease is a rare inherited pathology that causes cyst development in multiple organs, including the pancreas. 52 A

Comment on this QuestionOther complications include pneumothorax, dislodgement of the tube (partial or complete), obstruction of the tube or trachea, tracheal stenosis, sepsis, cuff prolapse, intubation granuloma of the vocal cords. S ubglottic stenosis occurs if the first and second tracheal rings are damaged.

102

A 54-year-old female undergoes a wide local excision of a basal cell carcinoma to her left nasolabial region. It is decided to proceed with a full-thickness skin graft to replace the defect.Which of the following sites would be the best donor site for this?

Select one answer only

Anterior forearm

Anterior thigh

Inguinal region

Postauricular area « YOUR ANSWERPosterior forearm

YOUR ANSWER WAS CORRECT The Answer

Although the forearm and thigh are good donor sites for split thickness skin grafting, they have limitations for facial reconstruction in comparison with other sites. Pre- and post-auricular areas are excellent for facial reconstruction as they are hairless and usually match skin colour and texture. Often several stages of reconstruction are required to obtain the best possible cosmetic result. If possible, facial soft tissue should be used to replace facial defects, hence forehead flaps are suitable for nasal reconstructions.In this case, wide local excision of the malignant lesion would necessitate a full-thickness skin graft. Such grafts include all skin layers down to the level of the dermal fat, and the donor defect must be primarily closed as no skin element remains in the donor bed.

Page 78: 1filedownload.com · Web view2014/12/19  · Von-Hippel Lindau disease is a rare inherited pathology that causes cyst development in multiple organs, including the pancreas. 52 A

The size and site of such grafts are therefore limited to areas where there are sufficient skin laxity to allow for primary closure (e.g. inguinal or wrist creases, and post-auricular regions). Full thickness grafts also tend to retain their colour and texture, and do not contract significantly.

103

The inferior sagittal sinus:

Single best answer question – choose ONE true option onlyDrains directly into the confluence of sinuses « YOUR ANSWERIs found in the falx cerebelli

Is formed within the free, inferior border of the falx cerebri « CORRECT ANSWER

Drains into the superior petrosal sinus

Contains valves, unlike the other venous dural sinuses

YOUR ANSWER WAS INCORRECT The Answer Comment on this Question

The inferior sagittal sinus is enclosed in the posterior half or two thirds of the free margin of the falx cerebri. It is cylindrical in shape. It increases in size as it passes backward and ends in the straight sinus. It receives several veins from the falx cerebri and occasionally receives a few veins from the medial surfaces of the hemispheres.

104

A 27-year-old male is seen in A&E after being admitted following an alleged assault on a night out. He is very inebriated, but examination reveals malocclusion of the teeth and an x-ray shows a displaced fracture of the mandible.It is thought he will require surgery, which of the following imaging modalities is best to plan reconstruction?

Select one answer only

Page 79: 1filedownload.com · Web view2014/12/19  · Von-Hippel Lindau disease is a rare inherited pathology that causes cyst development in multiple organs, including the pancreas. 52 A

CT « YOUR ANSWERFluoroscopy

Lateral X-rays

MRI

Occipitomental view (OM) X-ray

YOUR ANSWER WAS CORRECT The Answer Comment on this Question

Mandibular fractures are among the commonest facial fracture, given its prominence on the face. Fractures usually obey the ‘ring bone rule’, meaning that a fracture at one point usually indicates a corresponding fracture elsewhere. Approximately 50% of mandibular fractures are bilateral. Therefore multiple fractures are the norm.Malocclusion of the teeth commonly occurs, temporo-mandibular joint (TMJ) dysfunction occurs with condylar fractures. Computed tomography (CT) scanning is the imaging of choice after plain X-rays in planning reconstruction.

105

With regards to the lumbar spine anatomy.

Which part of the vertebra is responsible for causing spondylolysis?

Intervertebral disc

Lamina

Pars interarticularis « CORRECT ANSWERPedicle

Transverse process « YOUR ANSWER

YOUR ANSWER WAS INCORRECT The Answer Comment on this Question

Spondylolysis (i.e. ‘broken vertebra’) commonly refers to a defect at the pars interarticularis of the 5th (and less frequently, the 4th) lumbar vertebra, of

Page 80: 1filedownload.com · Web view2014/12/19  · Von-Hippel Lindau disease is a rare inherited pathology that causes cyst development in multiple organs, including the pancreas. 52 A

uncertain aetiology. On average, 5% of the population have a spondylolysis by the age of 5 years (this rises to 6% in adults). Patients are usually asymptomatic in the absence of slippage. Excessive stretching may cause a spondylolisthesis, resulting in deep lumbar pain. This is especially common in young and active adults (e.g. sportsmen who hyperextend their spine). Neurological signs are typically absent unless localised nerve root compression is present.

106

A 63-year-old female undergoes a subtotal thyroidectomy for a toxic multinodular goitre.Which of the following statements is most likely to be true?

Select one answer only

Damage to the external laryngeal nerve causes loss of low pitched phonation

The inferior thyroid artery arises from the origin of the external carotid artery

The inferior thyroid artery should be ligated as far laterally as possible « CORRECT ANSWER

The isthmus lies anterior to the thyroid cartilage

The superior thyroid artery enters the upper pole of the thyroid gland close to the recurrent laryngeal nerve« YOUR ANSWER

YOUR ANSWER WAS INCORRECT The Answer Comment on this Question

The superior thyroid artery arises from the external carotid artery and enters the upper pole of the thyroid gland close to the external laryngeal nerve, which supplies the cricothyroid muscle, a tensor of the vocal cord. Damage to this nerve causes the loss of high-pitched phonation.The inferior thyroid artery, absent in 5%, arises from the thyrocervical trunk of the subclavian artery. The inferior thyroid artery should be ligated as far laterally as possible to avoid damaging the recurrent laryngeal nerve. Damage to one recurrent laryngeal nerve causes a weakened voice, damage to both causes semiadduction and respiratory difficulties. The isthmus is normally in front of the second and third tracheal rings, although variations are common.

Page 81: 1filedownload.com · Web view2014/12/19  · Von-Hippel Lindau disease is a rare inherited pathology that causes cyst development in multiple organs, including the pancreas. 52 A

107

Postganglionic sympathetic fibres innervating the dilator pupillae muscle begin in the:

Single best answer question – choose ONE true option onlyCiliary ganglion « YOUR ANSWERSuperior cervical ganglion « CORRECT ANSWERBrain

Trigeminal ganglion

Spinal cord (T1-L2)

YOUR ANSWER WAS INCORRECT The Answer Comment on this Question

The cervical portion of the sympathetic trunk consists of three ganglia, named according to their positions as the superior, middle and inferior ganglia and connected by intervening cords. This portion receives no white rami communicantes from the cervical spinal nerves. Its spinal fibres are derived from the white rami of the upper thoracic nerves and enter the corresponding thoracic ganglia of the sympathetic trunk, through which they ascend into the neck. The superior cervical ganglion, the largest of the three, is located opposite the second and third cervical vertebrae. It is reddish-grey in colour and usually fusiform in shape. It is thought to be formed by the coalescence of four ganglia, corresponding to the upper four cervical nerves. It is related anteriorly to the sheath of the internal carotid artery and the internal jugular vein and posteriorly to the longus capitis muscle. It contains neurones that supply sympathetic innervation to the face (including the dilator pupillae muscle of the iris).

108

The nasolacrimal duct empties into the:

Single best answer question – choose ONE true option only

Page 82: 1filedownload.com · Web view2014/12/19  · Von-Hippel Lindau disease is a rare inherited pathology that causes cyst development in multiple organs, including the pancreas. 52 A

Sphenoethmoidal recess « YOUR ANSWERInferior meatus « CORRECT ANSWERMiddle meatus

Maxillary sinus

Infundibulum

YOUR ANSWER WAS INCORRECT The Answer Comment on this Question

The nasolacrimal duct is a membranous canal, about 18 mm in length, which extends from the lower part of the lacrimal sac to the inferior meatus of the nose, where it ends by a somewhat expanded orifice, provided with an imperfect valve, the plica lacrimalis (Hasner’s fold), formed by a fold of the mucous membrane. It is contained in an osseous canal, formed by the maxilla, the lacrimal bone and the inferior nasal concha. It is narrower in the middle than at either end and is directed downward, backward and a little lateralward. The mucous lining of the lacrimal sac and nasolacrimal duct is covered with columnar epithelium, which in places is ciliated.

109

You are examining a child in the ENT clinic whose mother says has had previous problems with his Eustachian tubes but has no other details.The Eustachian tube:

Can be obstructed by an enlarging palatine tonsil « YOUR ANSWERCloses during swallowing

Drains the inner ear

Gives attachment to the tensor veli palatini muscle « CORRECT ANSWERPierces the pharyngobasilar fascia to drain into the oropharynx

YOUR ANSWER WAS INCORRECT The Answer Comment on this Question

The pharyngotympanic tube drains the middle ear into the nasopharynx by piercing the pharyngobasilar fascia. The cartilaginous part gives attachment to

Page 83: 1filedownload.com · Web view2014/12/19  · Von-Hippel Lindau disease is a rare inherited pathology that causes cyst development in multiple organs, including the pancreas. 52 A

the tensor veli palatini muscle. It opens on swallowing under the action of the salpingopharyngeus and tensor palati muscles, and is obstructed by an enlarging pharyngeal tonsil (adenoids).

110

The foramen magnum is one of several openings at the base of the skull

Which important neurological structure/s pass through the foramen magnum?

Fascial nerve VII

Hypoglossal nerve XII

Optic nerve II

The medulla oblongata and the spinal accessory nerve « YOUR ANSWERVestibulocochlear nerve VIII

YOUR ANSWER WAS CORRECT The Answer Comment on this Question

The structures passing through the foramen magnum include: the medulla oblongata; meninges; spinal parts of the accessory nerves; meningeal branches of the upper cervical nerves; the vertebral arteries; and the anterior and posterior spinal arteries.

111

A 20-year-old schizophrenic man presents to a major trauma centre having been stabbed in the neck with a screwdriver. On assessment he has no catastrophic haemorrhage, nor difficulty breathing and a normal neurological examination. Before further assessment can be performed he discharged himself from hospital against the medical team’s advice. He presents to his GP four weeks later complaining of a loud buzzing on the same side of his head that he was stabbed on. 

Page 84: 1filedownload.com · Web view2014/12/19  · Von-Hippel Lindau disease is a rare inherited pathology that causes cyst development in multiple organs, including the pancreas. 52 A

What is the most likely cause of his symptoms?

This is likely to be a congenital arterio-venous fistula

This is likely to be a post traumatic arterio-venous fistula, which requires treatment surgery to excise the fistula and repair the artery and vein« YOUR ANSWER

This is likely to be a post traumatic arterio-venous fistula which will resolve without intervention and follow up should be arranged to review this in 3 months

This is likely to be an auditory hallucination related to his schizophrenia. He should be referred to the psychiatric services for review of his anti-psychotic medication

This is likely to be caused by a tracheo-oesophageal fistula, which requires surgical repair

YOUR ANSWER WAS CORRECT The Answer Comment on this Question

Arteriovenous fistulae in the neck commonly occur after penetrating trauma. The patient complains of loud buzzing or pulsating noise in the head on the side of the fistula. Dizziness may be a feature but neurological symptoms are usually absent. Carotid–jugular fistulae are easily amenable treatment by excision of the fistula and repair of the artery and vein. Vertebral fistulae are more difficult to access surgically and repair is not necessary as the vertebral artery may often be sacrificed with no danger of neurological sequelae.

Tracheo-oesophagael fistula (TOF) are usually congenital as a result of failure of fusion of the tracheoesophageal ridges during the third week of embryological development or the result of a surgical procedure such as a laryngectomy.

112

The transverse cervical artery is severed in a road traffic accident. Which muscle would be affected the most?

Single best answer question – choose ONE true option only

Page 85: 1filedownload.com · Web view2014/12/19  · Von-Hippel Lindau disease is a rare inherited pathology that causes cyst development in multiple organs, including the pancreas. 52 A

Levator scapulas « YOUR ANSWERRhomboideus minor

Rhomboideus major

Trapezius « CORRECT ANSWERLatissimus dorsi

YOUR ANSWER WAS INCORRECT The Answer Comment on this Question

The transverse cervical artery supplies blood to the trapezius. Levator scapulas and the rhomboids receive blood from the dorsal scapular artery. Latissimus dorsi receives blood from the thoracodorsal artery.

113

An 18-year-old female presented with ear-ache, a conductive deafness and a temperature of 39oC.

What is the most common cause? Paget’s disease of bone

Acoustic neuroma

Otosclerosis

A fracture through the petrous temporal bone

Otitis media « YOUR ANSWER

YOUR ANSWER WAS CORRECT The Answer Comment on this Question

The most common causes of conductive deafness include wax, acute otitis media, secretory otitis media, chronic otitis media, barotrauma, otosclerosis and injuries to the tympanic membrane and otitis externa. Other less common causes include tumours of the middle ear and traumatic ossicular dislocation. Sensory neural deafness is caused by a number of causes including infections such as mumps, herpes zoster, meningitis and syphilis. Other causes include congenital–maternal rubella, cytomegalovirus, toxoplasmosis, prolonged

Page 86: 1filedownload.com · Web view2014/12/19  · Von-Hippel Lindau disease is a rare inherited pathology that causes cyst development in multiple organs, including the pancreas. 52 A

exposure to loud noises, drugs (aspirin, aminoglycosides), Meniere’s disease, head injury and acoustic neuroma. Metabolic causes include diabetes and hypothyroidism. In Paget’s disease there may be a mixed hearing loss ie conduction and sensorineural deafness. This is due to direct involvement of the ossicles of the inner ear due to ankylosis of the stapes, or by impeachment of bone on the eighth cranial nerve in the auditory foramen.

114

A 15-year-old rugby player sustains a head injury during a match and collapses after initially playing on. He is taken to A+E where a CT scan demonstrates a large left extradural haematoma. At operation, damage to the middle meningeal artery is noted to be the cause.Which of the following is correct regarding this artery?

Select one answer only

Arises from the superficial temporal branch of the external carotid artery

Enters the skull through the foramen ovale

Is unlikely to be damaged in fractures at the pterion

Originates in the pterygopalatine fossa

Supplies part of the dura mater and the calvaria « YOUR ANSWER

YOUR ANSWER WAS CORRECT The Answer

The middle meningeal artery divides from the maxillary branch of the external carotid, entering the cranium through the foramen spinosum.

It is the largest artery that supplies the dura and also the calvaria. While the maxillary artery does give off branches in the pterygopalatine fossa region, the middle meningeal artery comes off more proximally, deep to the ramus of the mandible. It is particularly prone to damage caused by temporal bone fractures. The anterior division runs beneath the pterion, which is the junction of the parietal, frontal and squamous temporal bones with the greater wing of the sphenoid, and is prone to damage from fractures here.

Page 87: 1filedownload.com · Web view2014/12/19  · Von-Hippel Lindau disease is a rare inherited pathology that causes cyst development in multiple organs, including the pancreas. 52 A

115

Which of the following is most likely to cause a bitemporal hemianopia?

A craniopharyngioma

A pituitary adenoma « YOUR ANSWERA pituitary astrocytoma

A sphenoid wing meningioma

Pituitary apoplexy

YOUR ANSWER WAS CORRECT The Answer

The pituitary fossa, or sella turcica, lies above the body of the sphenoid bone and its associated sinus. The pituitary fossa can be easily identified on a lateral skull X-ray. The optic chiasm lies above and towards the back of the sella turcica.

A pituitary tumour would first impinge on the anterior part of the optic chiasm and so cause a temporal hemianopia.

Craniopharyngiomas comprise of 1.2-3% of all brain tumours in adults.

Although meningiomas can compress the optic chiasm, one based on the sphenoid wing would cause other symptoms prior to a hemianopia.

Pituitary tumours comprise between 10-15% of all intracranial tumours. Pituitary astrocytomas are incredibly rare pituitary tumours.

Pituitary apoplexy is caused by infarction of the pituitary gland caused by bleeding into a tumour, although it is a cause of bitemporal hemianopia it is less common than visual defects because of an adenoma.

116

A 22-year-old student is brought into A&E in severe pain around his jaw. On examination his mouth is wide open and he cannot close it. His friend reports

Page 88: 1filedownload.com · Web view2014/12/19  · Von-Hippel Lindau disease is a rare inherited pathology that causes cyst development in multiple organs, including the pancreas. 52 A

the pain developed whilst he was yawning and subsequently he has been like this. He is diagnosed as having a dislocated temporomandibular joint which is reduced in A&E.Which are the following muscles is most important in the active opening of the mouth?

Select one answer only

Buccinator

Lateral pterygoid « CORRECT ANSWERMasseter « YOUR ANSWERMedial pterygoid

Temporalis

YOUR ANSWER WAS INCORRECT The Answer Comment on this Question

The muscles of mastication – medial pterygoid, lateral pterygoid, masseter and temporalis all derive from the 1stbranchial arch and are supplied by the mandibular branch of the trigeminal nerve. Buccinator is a muscle of the cheek and aids in chewing by pressing the cheek against the molar teeth. It is not classed as a muscle of mastication though and is supplied by the facial nerve. Of the muscles of mastication the lateral pterygoid muscle arises by two heads: one from the infratemporal surface of the skull and the lower from the lateral surface of the lateral pterygoid plate. It is supplied by the anterior division of the mandibular nerve. It is indispensable to active opening of the mouth – drawing the condyle and disc forwards from the mandibular fossa. The second branchial arch forms the hyoid bone. The pterygoid venous plexus is a network of very small veins around and within the lateral pterygoid muscle. It drains into a pair of large short maxillary veins.

117

You are examining a patient in clinic who is referred by the GP with a swelling in the neck.The posterior triangle of the neck contains the:

Facial nerve

Great auricular nerve

Page 89: 1filedownload.com · Web view2014/12/19  · Von-Hippel Lindau disease is a rare inherited pathology that causes cyst development in multiple organs, including the pancreas. 52 A

Omohyoid muscle « YOUR ANSWERRoots of the brachial plexus

Vertebral artery

YOUR ANSWER WAS CORRECT The Answer Comment on this Question

The great auricular nerve turns upwards round the lateral border of the sternocleidomastoid muscle outside the triangle. The posterior belly of the omohyoid muscle and its intermediate tendon pass diagonally through the triangle (they are palpable, often confused with both the upper trunk of the brachial plexus and the suprascapular nerve), and the clavicular (intermediate) and acromial (lateral) branches of the supraclavicular nerves course through the roof. Lying medially, outside the triangle, are the roots of the brachial plexus, sandwiched between the scalenus anterior and medius muscles. The vertebral artery lies within the pyramidal space inferiorly, before ascending through the foramen transversarium of the C1–C6 vertebrae. Other components of the posterior triangle include the semispinalis capitis, splenius capitis, levator scapulae and scalenus medius muscles and the spinal accessory nerve.

118

You are reviewing a patient with a facial nerve palsy after resection of an acoustic neuroma.The facial nerve supplies the:

Buccinator muscle « YOUR ANSWERMedial pterygoid muscle

Parasympathetics to the parotid gland

Sensation to the inside of the mouth

Sympathetics to the lacrimal gland

YOUR ANSWER WAS CORRECT The Answer Comment on this Question

Page 90: 1filedownload.com · Web view2014/12/19  · Von-Hippel Lindau disease is a rare inherited pathology that causes cyst development in multiple organs, including the pancreas. 52 A

The facial nerve gives off a branch to the stapedius before it enters the stylomastoid foramen. The buccinator is supplied by the buccal branches of the facial nerve. The medial (and lateral) pterygoid is supplied by the mandibular division of the trigeminal nerve. The parasympathetic supply to the lacrimal gland is from the secretomotor fibres from the superior salivatory nucleus travelling in the intermediate and greater petrosal nerves and relaying in the pterygopalatine ganglion. The parasympathetic supply to the parotid gland is from the inferior salivatory nucleus via the tympanic and lesser petrosal branches of the glossopharyngeal nerve that project to the otic ganglion (preganglionic). Postganglionic fibres pass from the otic ganglion to the parotid via the auriculotemporal nerve.

119

The name of the bone that the internal carotid artery enters to reach the intracranial cavity is the:

Single best answer question – choose ONE true option onlyFrontal

Occipital

Sphenoid « YOUR ANSWERTemporal « CORRECT ANSWERMaxillary

YOUR ANSWER WAS INCORRECT The Answer Comment on this Question

The internal carotid artery supplies the anterior part of the brain, the eye and its appendages and the forehead and nose. In the adult it is the same size as the external carotid, but in the child it is larger than the external carotid. It is remarkable for the number of curvatures that it presents in different parts of its course. It occasionally has one or two curvatures near the base of the skull, while in its passage through the carotid canal in the petrous part of the temporal bone and along the side of the body of the sphenoid bone it describes a double curvature and resembles an italic letter ‘s’. The carotid canal is found on the inferior surface of the petrous part of the temporal bone. It ascends vertically at

Page 91: 1filedownload.com · Web view2014/12/19  · Von-Hippel Lindau disease is a rare inherited pathology that causes cyst development in multiple organs, including the pancreas. 52 A

first and then bends and runs horizontally forward and medially. It transmits the internal carotid artery and the carotid plexus of nerves into the cranium.

120

You are treating a patient who has sustained a stab wound to the face which has completely lacerated the mandibular branch of the trigeminal nerve at its origin.Which of the following muscles would remain unaffected?

Anterior belly of digastric

Lateral pterygoid

Masseter

Medial pterygoid

Posterior belly of digastric « YOUR ANSWER

YOUR ANSWER WAS CORRECT The Answer Comment on this Question

The mandibular division of the trigeminal nerve supplies all the muscles of mastication, including the lateral pterygoid, medial pterygoid, master and temporalis. It also supplies the anterior belly of digastric, mylohyoid, tensor tympani and tensor veli palatini.

121

One of your patients has a CT scan, which confirms a spinal tumour at the level of the bifurcation of the trachea. An image-guided biopsy is to be performed. 

Which spinal level corresponds to the bifurcation of the trachea?

Lower border C7 vertebra

Upper border T2 vertebra « YOUR ANSWERLower border T2 vertebra

Upper border T3 vertebra

Page 92: 1filedownload.com · Web view2014/12/19  · Von-Hippel Lindau disease is a rare inherited pathology that causes cyst development in multiple organs, including the pancreas. 52 A

Lower border T4 vertebra « CORRECT ANSWER

YOUR ANSWER WAS INCORRECT The Answer

The manubriosternal joint acts as a surface landmark for the tracheal bifurcation at T4. The cricoid cartilage is at the C6 level, while the soft palate may reach down as far as C3. The thyroid isthmus normally lies over the second and third tracheal rings and hence is a safe place for an emergency surgical airway. The common carotid artery bifurcates into the external and internal carotid arteries the upper border of the thyroid cartilage (C4).

122

You are reading a radiology report before the head and neck cancer MDT. A lesion is described as being on the 'True vocal cords' on an MRI of the neck.The true vocal cords are:

Abducted by the lateral cricoarytenoid muscles « YOUR ANSWERAbducted by the posterior cricoarytenoid muscles « CORRECT ANSWERFormed by the lower free edge of the quadrangular membranes

Innervated by sensory fibres of the internal laryngeal nerves

Lined by respiratory epithelium

YOUR ANSWER WAS INCORRECT The Answer

The true vocal folds have a stratified squamous epithelium, innervated by the recurrent laryngeal branch (CN X), and are formed by the vocal ligament (the free edge of the quadrangular membrane forms the false vocal cord). Above the vocal cords, the larynx is sensorily innervated by the internal laryngeal nerve (CN X).

The cords are adducted by the lateral cricoarytenoid muscle, abducted by the posterior cricoarytenoid

Page 93: 1filedownload.com · Web view2014/12/19  · Von-Hippel Lindau disease is a rare inherited pathology that causes cyst development in multiple organs, including the pancreas. 52 A

and tensed by tilting the thyroid cartilage downwards and forwards by contracting the cricothyroid muscle.

All the laryngeal muscles are supplied by the recurrent laryngeal nerve except for cricothyroid, which is supplied by the external laryngeal nerve.

123

The thyroid gland in some cases can have a thyroidea ima artery that supplies the isthmus of the thyroid. If present, it would take origin:

Single best answer question – choose ONE true option onlyFrom the inferior thyroid artery

Directly from the thyrocervical trunk « YOUR ANSWERFrom the superior thyroid artery

From the brachiocephalic trunk « CORRECT ANSWERDirectly from the external carotid artery

YOUR ANSWER WAS INCORRECT The Answer

The thyroidea ima artery, when present, arises from the brachiocephalic trunk (innominate artery) and ascends in front of the trachea to the lower part of the thyroid gland, which it supplies. It varies greatly in size and appears to compensate for deficiency or absence of one of the other thyroid vessels. It occasionally arises from the aorta, the right common carotid, the subclavian or the internal mammary.

124

A 72-year-old male presents with a swelling in the anterior triangle of the neck. Biopsy confirms a superficial submandibular gland carcinoma.Which structure is least likely to be involved?

Select one answer only

Deep cervical fascia

Page 94: 1filedownload.com · Web view2014/12/19  · Von-Hippel Lindau disease is a rare inherited pathology that causes cyst development in multiple organs, including the pancreas. 52 A

Platysma

The cervical branch of the facial nerve « YOUR ANSWERThe facial artery

The facial vein

YOUR ANSWER WAS CORRECT The Answer

The submandibular gland is a lobulated gland made up of a superficial and a deep part, which are continuous with each other around the posterior border of the mylohyoid muscle. Part of the gland lies infero-laterally, enclosed in an investing layer of deep cervical fascia, platysma muscle and skin. Laterally it is crossed by the cervical branch of the facial nerve and vein.The facial artery is related to the posterior and superior aspects of the superficial part of the gland. The marginal mandibular branch of the facial nerve passes beneath platysma and depressor angulioris in the proximity of the submandibular gland where it may be damaged during surgery.

125

A 29-year-old male presents to clinic concerning a lump on the posterior third of his tongue.Which of the following is this most likely to represent?

Select one answer only

Filiform papillae

Fungiform papillae « YOUR ANSWERLymphoid tissue « CORRECT ANSWERPalatine tonsil

Vallate papillae

YOUR ANSWER WAS INCORRECT The Answer

Page 95: 1filedownload.com · Web view2014/12/19  · Von-Hippel Lindau disease is a rare inherited pathology that causes cyst development in multiple organs, including the pancreas. 52 A

Three types of papillae are present on the upper surface of the anterior two-thirds of the tongue, namely: filiform, fungiform and (circum) vallate papillae. The mucous membrane covering the posterior one-third of the tongue is devoid of papillae but has a nodular irregular surface caused by the presence of underlying lymphatic tissue, the lingual tonsil. The palatine tonsils lie lateral to the tongue posterior to the palatoglossal arches.

126

A 60-year-old man arrives in A&E resus with a suspected odontoid peg fracture.The odontoid peg is:

Connected to the axis by the alar ligament

Part of the atlas (C1 vertebrae)

Seen on ‘open-mouth’ view X-ray « YOUR ANSWERSuperior to the notochord remnant

Weight bearing

YOUR ANSWER WAS CORRECT The Answer Comment on this Question

The axis (C2 vertebra) is characterised by the odontoid peg or dens that articulates with the atlas. It bears no weight. The alar ligaments together with the apical ligament are attached from the sloping upper edge of the odontoid peg to the margins of the foramen magnum. The alar ligaments limit rotation of the head and are very strong. The apical ligament is the fibrous remnant of the notochord and this lies superior to the odontoid peg.

127

You are in the ENT clinic reviewing a patient with chronic infection of the middle ear, otitis media.The middle ear:

Contains all the auditory ossicles « YOUR ANSWERHas a promontory in the projection of the lateral semicircular canal

Page 96: 1filedownload.com · Web view2014/12/19  · Von-Hippel Lindau disease is a rare inherited pathology that causes cyst development in multiple organs, including the pancreas. 52 A

Has the facial nerve running through its roof

Has the internal carotid artery running medially and posteriorly

Has the internal jugular vein running medially and superiorly

YOUR ANSWER WAS CORRECT The Answer Comment on this Question

The medial wall, which separates the middle from the internal ear, contains the oval window and promontory. The promontory is the rounded projection of the first turn of the cochlea. Also medially, the internal auditory meatus carries the facial nerve. The floor is a thin bone separating the middle ear from the bulb of the jugular vein. Anteriorly, a thin bony wall separates the cavity from the internal carotid artery.

128

A worried mum has brought her 4-year-old son to the GP because she is concerned about his speech which she has noticed has deteriorated. On further questioning it emerges he has some difficulty hearing instructions and has to turn the volume up very high when watching television. His only past medical history of note is recurrent upper respiratory tract infections.What is the most likely cause for his symptoms?

Select one answer only

Cholesteatoma

Glue ear « CORRECT ANSWERMastoiditis

Otosclerosis « YOUR ANSWERPresbyacusis

YOUR ANSWER WAS INCORRECT The Answer Comment on this Question

Many of the problems resulting from glue ear are secondary to hearing loss. In children this can manifest in several different ways such as problems with

Page 97: 1filedownload.com · Web view2014/12/19  · Von-Hippel Lindau disease is a rare inherited pathology that causes cyst development in multiple organs, including the pancreas. 52 A

speech and language, not hearing instructions or turning up the volume on the television. Glue ear (otitis media with effusion) is the most common complication of acute otitis media.Glue ear is the commonest cause of hearing loss in young children. Enlarged adenoids, common in children, may occlude the Eustachian tube orifice. The palatal muscles help to open the Eustachian tube, so glue ear is common in children with a cleft palate. There is evidence confirming that parental smoking increases the development of glue ear.

129

Itching sensation from the skin immediately over the base of the spine of your scapula is mediated through the:

Single best answer question – choose ONE true option onlyAccessory nerve « YOUR ANSWERDorsal primary ramus of T2 « CORRECT ANSWERDorsal root of T2

Ventral primary ramus of T2

Ventral root of T2

YOUR ANSWER WAS INCORRECT The Answer Comment on this Question

Dorsal and ventral primary rami are the first branches off spinal nerves. Dorsal rami provide sensory innervation to the skin over the back and give motor innervation to the true back muscles; ventral rami supply sensory innervation to the skin over the limbs and the skin over the ventral side of the trunk. Ventral rami also give motor innervation to the skeletal muscles of the neck, trunk and extremities. The accessory nerve, which innervates the trapezius, is not responsible for any sensory innervation. The dorsal and ventral roots of spinal nerves are not directly responsible for any sensory innervation to the skin. Dorsal and ventral rootlets emerge from the spinal cord to form the dorsal and ventral roots. The ventral roots contain efferent motor fibres to skeletal muscles,

Page 98: 1filedownload.com · Web view2014/12/19  · Von-Hippel Lindau disease is a rare inherited pathology that causes cyst development in multiple organs, including the pancreas. 52 A

while the dorsal roots contain afferent sensory fibres. These roots combine to form the spinal nerve, which then gives off the primary rami.

130

Of the following intrinsic muscles of the larynx, which tenses (stretches) the vocal folds?

Single best answer question – choose ONE true option onlyPosterior cricoarytenoid

Lateral cricoarytenoid

Thyroarytenoid

Transverse arytenoid

Cricothyroid muscle « YOUR ANSWER

YOUR ANSWER WAS CORRECT The Answer Comment on this Question

The cricothyroid, triangular in form, arises from the front and lateral part of the cricoid cartilage; its fibres diverge and are arranged in two groups. The lower fibres constitute a pars obliqua and slant backward and lateralward to the anterior border of the inferior cornu. The anterior fibres, forming pars recta, run upward, backward and lateralward to the posterior part of the lower border of the lamina of the thyroid cartilage. The external laryngeal branch of the superior laryngeal nerve supplies the cricothyroid. The cricothyroids produce tension and elongation of the vocal folds by drawing up the arch of the cricoid cartilage and tilting back the upper border of its lamina. The distance between the vocal processes and the angle of the thyroid is so increased and the folds are consequently elongated.

131

Following an RTA, a 35-year-old woman, who had not been wearing a seat belt is brought into A&E with a clear nasal discharge.

Page 99: 1filedownload.com · Web view2014/12/19  · Von-Hippel Lindau disease is a rare inherited pathology that causes cyst development in multiple organs, including the pancreas. 52 A

Which type of fracture predisposes the patient to CSF rhinorrhea?

Le Fort I fracture due to mobility of teeth

Le Fort II due to fracture of the orbital floor

Le Fort III due to fracture of the ethmoid « CORRECT ANSWERLe Fort IV as it involves the frontal bone

Le Fort maxillary fractures are not associated with CSF rhinorrhea « YOUR ANSWER

YOUR ANSWER WAS INCORRECT The Answer Comment on this Question

The French surgeon Le Fort performed experiments on cadavers in the early 1900s and classified facial fractures into I, II and III. In Le Fort I injuries the fracture line passes above the palate, fracturing the pyrimadal processes of the maxilla on each side, the vomer and the lower parts of the pterygoid processes. Bilateral Le Fort I fractures represent 23% of these injuries, unilateral 11%. Le Fort I and II fractures occur together in 21% of cases. Le Fort III injuries involve the ethmoids and so increase intercanthal distance. Nasopharyngeal intubation is preferred to oropharyngeal intubation to secure the airway.

132

There are three main foramina: the superior and inferior orbital fissures, and the optic canal.

Which of the following enter the orbit via the optic canal? Abducens nerve

Lacrimal nerve

Ophthalmic artery « YOUR ANSWERSuperior and Inferior ophthalmic veins

Trochlear nerve

YOUR ANSWER WAS CORRECT The Answer

Page 100: 1filedownload.com · Web view2014/12/19  · Von-Hippel Lindau disease is a rare inherited pathology that causes cyst development in multiple organs, including the pancreas. 52 A

The optic nerve and ophthalmic artery pass through the optic canal. The optic nerve and retina are part of the central nervous system.

The optic nerve is invested by all the meningeal layers. Since the central retinal artery travels in the optic nerve after branching off the ophthalmic artery, damage to the optic nerve commonly causes retinal infarction. The other options in this question pass through the superior orbital fissure.

133

Which of the following effects is most likely to be seen if the right dorsal scapular nerve is injured near its origin?

Single best answer question – choose ONE true option onlySkin of the upper back on the right side would be numb

The point of the right shoulder would droop

Scapular retraction on the right would be weakened « YOUR ANSWERExtension of the right arm would be weakened

Inability to adduct the right arm

YOUR ANSWER WAS CORRECT The Answer Comment on this Question

The dorsal scapular nerve is a motor nerve off the C5 nerve root that innervates the rhomboids and levator scapulas. These muscles help to retract and elevate the scapula, so these movements would be weakened following that damage. The skin of the upper back on the right side is innervated by the dorsal primary rami of a spinal nerve. The point of the right shoulder, the acromion, is elevated by the trapezius. The trapezius is innervated by the accessory nerve, so the point of the shoulder would droop if the accessory nerve was damaged. Latissimus dorsi, innervated by the thoracodorsal nerve, allows for extension and adduction of the arm.

Page 101: 1filedownload.com · Web view2014/12/19  · Von-Hippel Lindau disease is a rare inherited pathology that causes cyst development in multiple organs, including the pancreas. 52 A

134

A 24-year-old male is rushed into A&E after being kicked in the neck during a karate competition. He has a hoarse voice and is very short of breath. On examination, there is a marked stutter, laryngeal tenderness, subcutaneous emphysema and the patient is drooling saliva.What is the first line treatment?

Nasopharyngeal tube insertion

CT angiogram of neck

Emergency tracheostomy

Endotracheal intubation with C-spine immobilisation « CORRECT ANSWERSurgical cricothyroidotomy « YOUR ANSWER

YOUR ANSWER WAS INCORRECT The Answer

Fracture of the larynx can present with acute airway obstruction. The mechanism of injury usually involves either blunt or penetrating trauma to the larynx. Although the larynx is protected by the mandible above and the sternum below, it may be crushed, between these two points, by a blunt object anteriorly and the cervical spine posteriorly. It is diagnosed by the following triad:

hoarseness of voice, subcutaneous emphysema and a palpable fracture.

Other signs and symptoms include stridor, dysphagia, haemoptysis, laryngeal tenderness, odynophagia and anterior neck pain. Furthermore, there may be loss of thyroid cartilage prominence and ecchymosis in the overlying skin. If the patient’s airway is totally obstructed or the patient is in severe respiratory distress, an attempt at endotracheal intubation is warranted (note that in trauma, C-spine immobilisation should be performed together with securing the airway). Flexible endoscopic-guided intubation may be helpful in this situation, but only if it can be performed promptly and by an experienced anaesthetist or surgeon. Surgical cricothyroidotomy, although not preferred, may be a lifesaving option. If intubation is unsuccessful and if surgical cricothyroidotomy cannot be performed, then an emergency tracheostomy is indicated. However, in

Page 102: 1filedownload.com · Web view2014/12/19  · Von-Hippel Lindau disease is a rare inherited pathology that causes cyst development in multiple organs, including the pancreas. 52 A

emergency conditions, tracheostomy is difficult to perform, may be associated with profound bleeding and may be time consuming. Computed tomography is the investigation of choice in suspected laryngeal trauma. It is especially useful in evaluating the extent of injury when flexible fibre optic examination is limited by laryngeal oedema.

135

A 26-year-old female suffers with chronic submandibular gland infections and undergoes an excision of the gland.Which of the following is a potential complication of surgery?

Select one answer only

Anaesthesia of the contralateral half of the tongue

Damage to the retromandibular artery

Deviation of the tongue to the contralateral side on protrusion

Frey’s syndrome

Weakness of the angle of the mouth « YOUR ANSWER

YOUR ANSWER WAS CORRECT The Answer

The structures to be avoided at the time of submandibular gland excision include:

1. the mandibular branch of the facial nerve; 2. the hypoglossal and lingual nerves; 3. the facial artery 4. and the retromandibular vein.

Facial nerve damage causes weakness of the angle of the mouth.

Lingual nerve damage results in ipsilateral anaesthesia and hemiplegia of the tongue.

Hypoglossal nerve injury restricts the mobility of the tongue with deviation to the ipsilateral side on protrusion.

Page 103: 1filedownload.com · Web view2014/12/19  · Von-Hippel Lindau disease is a rare inherited pathology that causes cyst development in multiple organs, including the pancreas. 52 A

Frey’s syndrome (gustatory sweating) is a complication of parotid surgery.

136

You are performing the approach for a thyroidectomy.Which of the following is correct regarding this approach?

The ansa cervicalis is seen readily

The anterior jugular veins should be divided « CORRECT ANSWERThe flaps raised should be superficial to platysma

The incision is made vertically at the level of the cricoid

The strap muscles are commonly divided « YOUR ANSWER

YOUR ANSWER WAS INCORRECT The Answer

The incision in thyroidectomy is made following the skin creases at about 4cm above the sternoclavicular joints.

This includes skin, subcutaneous tissue and the platysma muscle.

The anterior jugular veins, lying in the plane between the platysma are divided. The majority of surgeons rarely divide strap muscles (namely the sternothyroid muscle at its insertion on the thyroid cartilage) in order to gain exposure to the superior pole during thyroidectomy.The ansa cervicalis embedded in the carotid sheath is not usually sought, unlike the recurrent laryngeal nerves.

137

You are assisting your neurosurgical consultant performing an anterior cervical discectomy. He is concerned about injuring the recurrent laryngeal nerve.Which of the following is correct regarding the right recurrent laryngeal nerve?Single best answer - select one answer only

Arises from a cranial nerve passing through the foramen magnum

Is related to the inferior thyroid artery « YOUR ANSWER

Page 104: 1filedownload.com · Web view2014/12/19  · Von-Hippel Lindau disease is a rare inherited pathology that causes cyst development in multiple organs, including the pancreas. 52 A

Passes behind the ligamentum arteriosum

Runs posterior to the oesophagus but anterior to the trachea

Winds around the aortic arch

YOUR ANSWER WAS CORRECT The Answer Comment on this Question

The recurrent laryngeal nerves arise from the vagus. On the right, the recurrent laryngeal nerve winds around the subclavian artery; on the left, it winds around the aortic arch, passing behind the ligamentum arteriosum. Both nerves run in a groove between the trachea and oesophagus and are closely related to the inferior thyroid artery.

138

A 45-year-old woman is referred to the neurosurgical outpatient clinic by her GP after she complained of gradual loss of hearing in her left ear, with associated tinnitus and facial weakness. 

What is the most likely site of the lesion?

The foramen lacerum

The foramen magnum

The foramen rotundum

The foramen spinosum

The internal auditory canal « YOUR ANSWER

YOUR ANSWER WAS CORRECT The Answer Comment on this Question

This patient has presented with symptoms consistent with a vestibular schwannoma also known as an acoustic neuroma. These are benign primary intracranial tumours arising from the myelin forming cells of the vestibulo-cochlear (CN V111) nerve.

The foramen magnum transmits the vertebral arteries, which unite at the lower border

Page 105: 1filedownload.com · Web view2014/12/19  · Von-Hippel Lindau disease is a rare inherited pathology that causes cyst development in multiple organs, including the pancreas. 52 A

of the pons to form the basilar artery.

The foramen spinosum transmits the middle meningeal vessels and the meningeal branch of the mandibular nerve. 

The foramen rotundum contains the maxillary nerve. 

The foramen ovale transmits the mandibular nerve, lesser petrosal nerve and accessory meningeal artery. The foramen lacerum transmits the internal carotid and greater petrosal nerve, which leaves as a nerve of the pterygoid canal.

139

A 14-year-old boy presents to the Accident and Emergency department with erythematous, swollen tonsils and left ear pain. Examination of the ear is unremarkable.Referred pain from which nerve is most likely to be responsible for these symptoms?

Select one answer only

Facial nerve

Glossopharyngeal nerve « CORRECT ANSWERHypoglossal nerve

Lesser palatine nerve « YOUR ANSWERSuperior laryngeal nerve

YOUR ANSWER WAS INCORRECT The Answer Comment on this Question

Earache from acute tonsillitis is due to referred pain along the glossopharyngeal nerve, which runs in the tonsillar bed and sends a tympanic branch to the tympanic plexus in the middle ear. The other nerves are not involved.

140

Page 106: 1filedownload.com · Web view2014/12/19  · Von-Hippel Lindau disease is a rare inherited pathology that causes cyst development in multiple organs, including the pancreas. 52 A

Which of the following statements is CORRECT regarding the muscles of facial expression?

Single best answer question – choose ONE true option onlyThe digastric muscle is a muscle of facial expression

Some muscles of facial expression are innervated by the long buccal nerve « YOUR ANSWER

They are in the same subcutaneous plane as the platysma muscle « CORRECT ANSWER

Some muscles of facial expression receive their motor supply via the zygomaticofacial nerve

All muscles of facial expression are attached to bone

YOUR ANSWER WAS INCORRECT The Answer Comment on this Question

The facial muscles are subcutaneous (just under the skin, in the same plane as the platysma) muscles that control facial expression. They generally originate on bone and insert on the skin of the face. The facial muscles are innervated by cranial nerve VII, also known as the facial nerve. The facial muscles are derived from the second pharyngeal arch.

141

Following a left carotid endarterectomy a 74-year-old patient is noted to have developed speech problems. He seems able to understand what is said but unable to get the words out.Which area of the brain is most likely to be affected?

Basal ganglia

Broca’s area « YOUR ANSWEROccipital cortex

Olfactory cortex

Wernicke’s area

Page 107: 1filedownload.com · Web view2014/12/19  · Von-Hippel Lindau disease is a rare inherited pathology that causes cyst development in multiple organs, including the pancreas. 52 A

YOUR ANSWER WAS CORRECT The Answer

The primary motor cortex is in the precentralgyrus and the primary sensory cortex in the postcentral cortex. Broca’s area lies in the posterior part of the inferior frontal gyrus of the dominant hemisphere. Lesions to Broca’s area cause expressive dysphasia, classically described as knowing what to say, but being unable to get the words out. The putamen and globuspallidus are together known as the lentiform nucleus and form part of the basal ganglia. The olfactory impulses travel to the temporal lobe in the region of the uncus.

142

In head anatomy, the middle meningeal artery could be ruptured in head trauma.

Which of the following is true?

It is a branch of the maxillary artery which is a branch of the internal carotid artery

It branches off the carotid artery in the infratemporal fossa

It supplies part of the dura mater, periosteum and skull bones

When it ruptures in head injury it causes extradural haematoma « YOUR ANSWER

It passes through the foramen ovale

YOUR ANSWER WAS CORRECT The Answer Comment on this Question

The middle meningeal artery is a branch of the maxillary artery in the infratemporal fossa. The maxillary artery enters the infratemporal fossa before running forwards between the heads of lateral pterygoid passing into the pterygopalatine fossa. The middle meningeal artery passes through the foramen spinosum.

The dura mater is supplied by the anterior, middle and posterior meningeal arteries.

Page 108: 1filedownload.com · Web view2014/12/19  · Von-Hippel Lindau disease is a rare inherited pathology that causes cyst development in multiple organs, including the pancreas. 52 A

143

A dental surgeon carries out a block of the inferior alveolar nerve by infiltrating local anaesthetic at the mandibular foramen.Which clinical feature may result from this procedure? Single best answer question – choose ONE true option only

Numbness of the lower lip on the injected side « YOUR ANSWERIneffective block for the incisor teeth

Numbness of hard palate

Inability of the patient to clench his jaws

Transient weakness of the facial muscles on the injected side

YOUR ANSWER WAS CORRECT The Answer

The inferior alveolar nerve, a branch of the mandibular division of the trigeminal nerve (V), traverses the inferior alveolar, or dental, canal of the mandible to supply all the teeth of that hemimandible; all the teeth on that side are therefore anaesthetised. The mental branch of the nerve emerges through the mental foramen to supply the lower lip, which becomes numb in a successfully performed block. Note that although the inferior alveolar nerve is not associated with tongue sensation, administration of anaesthesia near the mandibular foramen partially anaesthetizes the nearby lingual nerve by diffusion, thus resulting in ipsilateral numbness to the side of the tongue.The muscles of the tongue, of mastication and of facial expression are not affected.

144

A 21-year-old male is rushed into resus after being stabbed in the back whilst on a night out. The injury is shown on MRI to involve a right hemisection of his spinal cord.If the spinothalamic tract is transected in the injury, what deficit will it cause below the lesion?

Loss of fine touch and proprioception on the right side

Page 109: 1filedownload.com · Web view2014/12/19  · Von-Hippel Lindau disease is a rare inherited pathology that causes cyst development in multiple organs, including the pancreas. 52 A

Loss of fine touch and propriception on the left side

Loss of movement on the right side

Loss of pain and temperature on the right side

Loss of pain and temperature on the left side « YOUR ANSWER

YOUR ANSWER WAS CORRECT The Answer

The spinothalamic tract conveys pain, temperature, touch and pressure sensations from one side of the body to the opposite side of the brain.

Vibration and position sense are conveyed via the posterior column.

The first neurone of the spinothalamic tract synapses in the posterior horn; the next neurone crosses to the right side of the spinal cord and synapse in the thalamus, after ascending through the cord and brainstem; the third neurone arises in the thalamus to pass to the cortex. The secondary axons of the spinothalamic tract ascend through the brainstem to synapse in the thalamus. Axons from the cervical region synapse medially while axons from the lumbar region synapse laterally.

A lesion of the spinothalamic tract anywhere in the brainstem would lead to a loss of pain sensations from the opposite side of the body. Temperature and touch sensations would also be diminished from the opposite side of the body but not totally lost because other pathways may also convey these modalities. A lesion of the spinothalamic tract at the level of the spinal cord would lead to loss of pain sensations on the opposite side, beginning one level below the level of the lesion.

145

A 71-year-old heavy smoker undergoes a Carotid endarterectomy for a recent transient ischaemic attack (TIA). After careful dissection anterior to the sternocleidomastoid muscle the carotid sheath is reached.Which of the following is the most anterior structure within the carotid sheath?

Select one answer only

Page 110: 1filedownload.com · Web view2014/12/19  · Von-Hippel Lindau disease is a rare inherited pathology that causes cyst development in multiple organs, including the pancreas. 52 A

Ansa Cervicalis

Common carotid artery

Internal carotid artery

Internal jugular vein « YOUR ANSWERVagus nerve

YOUR ANSWER WAS CORRECT The Answer Comment on this Question

The internal jugular vein descends the neck in the anterior aspect of the carotid sheath, and drains into the brachiocephalic vein on each side behind the sternoclavicular joint. The Vagus nerve passes vertically down within the Carotid sheath closely related to the internal carotid artery and lying between it and the internal jugular vein. The ansa loops anteriorly around the carotid sheath under cover of the sternocleidomastoid muscle.

146

Transection of the anterior division of the mandibular nerve (CN Vc) in the infra-temporal fossa is most likely to result in which one of the following?Select one answer only

Deviation of the jaw away from the side of the lesion on protrusion

Dysphagia

Ipsilateral anaesthesia of the mandibular teeth

Ipsilateral anaesthesia of the mucosa of the oral vestibule « YOUR ANSWERIpsilateral paralysis of the buccinator muscle

YOUR ANSWER WAS CORRECT The Answer Comment on this Question

The anterior division of CN Vc has one sensory branch (the buccal nerve to the skin of the cheek and mucosa of the vestibule). All other branches are motor to the muscles of mastication (the masseter, temporalis and lateral pterygoid). The

Page 111: 1filedownload.com · Web view2014/12/19  · Von-Hippel Lindau disease is a rare inherited pathology that causes cyst development in multiple organs, including the pancreas. 52 A

lower jaw teeth are supplied by the inferior alveolar branch of the posterior division of CN Vc.The buccinator muscle is supplied by the VIIth cranial nerve. Unopposed contraction of the contralateral lateral pterygoid muscle deviates the jaw to the side of the lesion during protrusion. None of the muscles of the pharynx are supplied by the anterior division of CN Vc, so dysphagia is not a feature of damage to this nerve.

147

The anterior approach to the cervical spine is indicated to remove tumours and fuse the spine. The platysma forms the most superficial plane.

Which nerve supplies the platysma?

Facial nerve « YOUR ANSWERRecurrent laryngeal nerve

Spinal accessory nerve

Sympathetic chain

Trigeminal nerve

YOUR ANSWER WAS CORRECT The Answer Comment on this Question

Seventy-five per cent of cases of acute facial nerve palsy are of unknown aetiology (Bell’s palsy); hence computed tomography (CT) scanning is not usually required in the presence of few clinical signs. The House–Brackmann scale of I (normal) to IV (complete paralysis) gives an indication of severity, with eye closure the most important observation as this can lead to corneal ulceration (grade III and above). Marginal mandibular nerve damage in submandibular gland surgery manifests as drooping of the corner of the mouth. Palsy following superficial parotidectomy is in the region of 0.5%.

148

Page 112: 1filedownload.com · Web view2014/12/19  · Von-Hippel Lindau disease is a rare inherited pathology that causes cyst development in multiple organs, including the pancreas. 52 A

You are assisting in theatre, performing a median thoracotomy when the subclavian artery is accidently damaged near its origin. Your foundation year doctor asks if you can simply tie off and ligate the artery to stop the bleeding. 

What is the most appropriate answer?

It depends on which side. The artery can be sacrificed on the left but not the right

No, the subclavian artery supplies the thyroid, breast and upper lobe of the lung

No, the subclavian artery supplies the thyroid gland, breast, rectus abdominus, brainstem and diaphragm« YOUR ANSWER

No. Unless pre-operative angiography has been performed to verify the anatomy. In 50% of the population the subclavian supplies the brain stem and in these people it cannot be sacrificed

Yes. The subclavian is a relatively minor artery and can be sacrificed with impunity

YOUR ANSWER WAS CORRECT The Answer

The thyroid gland is supplied by the inferior thyroid artery, a branch of the thyocervical trunk.

The internal thoracic artery supplies: the breast, through anterior intercostal vessels, usually in the second and third intercostal spaces; the rectus abdominis muscle, through the superior epigastric branch; and the diaphragm, through the musculophrenic artery. The vertebral arteries supply the brainstem through the posterior inferior cerebellar arteries.

149

During a superficial parotidectomy the facial nerve is inadvertently injured.Which one of the following problems might the patient experience?

Dry mouth

Loss of corneal reflex « YOUR ANSWER

Page 113: 1filedownload.com · Web view2014/12/19  · Von-Hippel Lindau disease is a rare inherited pathology that causes cyst development in multiple organs, including the pancreas. 52 A

Loss of taste from anterior two thirds of tongue

Loss of taste from posterior third of tongue

Numbness of cheek

YOUR ANSWER WAS CORRECT The Answer

Complications of parotidectomy are

1. haematoma formation, 2. wound infection, 3. greater auricular nerve injury (numbness of the earlobe and angle of the

jaw), 4. facial nerve palsy, 5. gustatory sweating (Frey's syndrome) 6. and a salivary fistula.

Because of the presence of other major and minor salivary glands, a dry mouth is not a commonly encountered problem. The efferent arm of the corneal reflex involves the facial nerve and so the corneal reflex may be affected (note that the afferent arm of the corneal reflex is through the trigeminal nerve). It is important to note that taste fibres to the anterior two thirds of the tongue extend from the chorda tympani via the lingual nerve and are less likely to be damaged during superficial parotidectomy.

150

A 25-year-old male presents with significant facial trauma following a road traffic collision with proptosis of the right eye. CT scan demonstrates a fracture across the superior orbital fissure with associated haematoma.Which nerve is least likely to be affected?

Select one answer only

The frontal nerve

The inferior division of the oculomotor nerve

The lacrimal nerve

Page 114: 1filedownload.com · Web view2014/12/19  · Von-Hippel Lindau disease is a rare inherited pathology that causes cyst development in multiple organs, including the pancreas. 52 A

The nasociliary nerve « YOUR ANSWERThe zygomatic nerve « CORRECT ANSWER

YOUR ANSWER WAS INCORRECT The Answer Comment on this Question

The superior orbital fissure lies at the apex of the orbit bounded primarily by the greater and lesser wings of the sphenoid bone. The tendons of the lateral rectus divide the fissure into superior and inferior parts. The trochlear, frontal and lacrimal nerves pass superiorly with the superior orbital vein. The superior and inferior branches of the oculomotor nerve, abducens and nasociliary nerves pass inferiorly with the inferior orbital vein.

The frontal and nasociliary nerves arise from the ophthalmic division of the trigeminal nerve in the lateral wall of the cavernous sinus. After exiting the superior orbital fissure, the frontal nerve divides into the supratrochlear and supraorbital nerves at the orbital margin to supply the skin of the forehead.The nasociliary nerve supplies the ethmoidal sinuses, sphenoidal sinuses, skin of the upper eyelids and nose. The inferior ramus of the oculomotor nerve gives off branches to the inferior rectus, medial rectus and the inferior oblique muscles. The superior ramus of the oculomotor nerves supplies the levatorpalpebraesuperioris and so may give rise to a ptosis if cut. The zygomatic nerve is a branch of the maxillary division of the trigeminal nerve which enters the orbit through the inferior orbital fissure.

151

A 78-year-old male taking warfarin experiences a sudden onset of headache with visual disturbance. CT scan demonstrates a left occipital lobe intracerebral haematoma.What visual finding would expect to find on examination?

Select one answer only

Bitemporal hemianopia

Central scotoma

Page 115: 1filedownload.com · Web view2014/12/19  · Von-Hippel Lindau disease is a rare inherited pathology that causes cyst development in multiple organs, including the pancreas. 52 A

Left homonymous hemianopia

Macular sparing always

Right visual field loss « YOUR ANSWER

YOUR ANSWER WAS CORRECT The Answer

A right homonymous hemianopia would result from a left occipital lobe infarction and the macular would also be affected. Macular or central field sparing can occur if the occipital pole remains intact through blood supply from a branch of the middle cerebral artery.

A central scotoma describes visual loss corresponding with the point of fixation most commonly caused by a lesion between the optic chiasm and the retina.

A bitemporal hemianopia is a sign of optic chiasm compression, oftendue to pituitary gland tumours.

152

Which of the following statements regarding the spinal arachnoid mater is CORRECT?

Single best answer question – choose ONE true option onlyIt is very thick compared with the cranial part

It is separated from the dura by the subarachnoid space

It forms the denticulate ligaments

It invests the cauda equina « CORRECT ANSWERIt has a rich plexus of nerves derived from the vagus « YOUR ANSWER

YOUR ANSWER WAS INCORRECT The Answer Comment on this Question

The spinal part of the arachnoid is a thin, delicate, tubular membrane loosely investing the spinal cord.

Page 116: 1filedownload.com · Web view2014/12/19  · Von-Hippel Lindau disease is a rare inherited pathology that causes cyst development in multiple organs, including the pancreas. 52 A

Above, it is continuous with the cranial arachnoid. Below, it widens out and invests the cauda equina and the nerves proceeding from it. It is separated from the dura mater by the subdural space, but here and there this space is traversed by isolated connective-tissue trabeculae, which are most numerous on the posterior surface of the spinal cord. The arachnoid surrounds the cranial and spinal nerves and encloses them in loose sheaths as far as their points of exit from the skull and vertebral canal. The arachnoid consists of bundles of white fibrous and elastic tissue intimately blended together. Its outer surface is covered with a layer of low cuboidal mesothelium. The inner surface and the trabeculae are likewise covered by a somewhat low type of cuboidal mesothelium, which in places is flattened to a pavement type. Vessels of considerable size, but few in number, and a rich plexus of nerves derived from the motor root of the trigeminal, the facial and the accessory nerves, are found in the arachnoid.

153

A 74-year-old man is noted to have a hoarse voice following a right hemithyroidectomy.Which nerve is most likely to have been injured?

Ansacervicalis

Glossopharayngeal nerve

Hypoglossal nerve

Recurrent laryngeal nerve « YOUR ANSWERSuperior laryngeal nerve

YOUR ANSWER WAS CORRECT The Answer Comment on this Question

Injury to the right recurrent laryngeal nerve is a major complication of a right hemithyroidectomy. The recurrent laryngeal nerves both ascend and lie posterior to the pretracheal fascia next to the medial surface of the thyroid lobes. The upper root of the ansa cervicalis runs anterior to the internal and common carotid arteries and joins the lower root. The glossopharyngeal nerve lies between the internal jugular vein and internal carotid artery. The hypoglossal nerve lies lateral to the occipital, internal carotid, external carotid and lingual

Page 117: 1filedownload.com · Web view2014/12/19  · Von-Hippel Lindau disease is a rare inherited pathology that causes cyst development in multiple organs, including the pancreas. 52 A

arteries and passes over the hyoid. The superior laryngeal nerve lies posterior, then medial to the internal carotid, then pierces the carotid sheath to run on the wall of the pharynx.

154

A 50-year-old man who had suffered long-standingback pain presents to the accident and emergency department with a sudden deterioration and pain radiating to the lateral malleolus. 

What is the most likely cause of these symptoms?

A prolapsed intervertebral disc at the L4/5 interspace

A prolapsed intervertebral disc at he L5/S1 interspace « YOUR ANSWERA prolapsed intervertebral disc compressing the cauda equina

A prolapsed intervertebral disc compressing the sacral segments of the cord

Collapse of a vertebral body secondary to osteoporosis with subsequent compression of the spinal canal

YOUR ANSWER WAS CORRECT The Answer

A prolapsed L5–S1 disc presses on the S1 spinal nerve (the L5 nerve passes above the prolapsed disc in the intervertebral foramen and so escapes damage). At the level of prolapse, the spinal canal contains the cauda equina and not cord per se. The S1 dermatome lies over the lateral malleolus. Exaggerated reflexes are diagnostic of an upper motor neurone lesion.

The S2 dermatome occupies the posterior aspect of the calf.Osteoporosis can cause painful collapse of vertebral bodies but rarely causes neurological symptoms and most often affects older women.

155

Page 118: 1filedownload.com · Web view2014/12/19  · Von-Hippel Lindau disease is a rare inherited pathology that causes cyst development in multiple organs, including the pancreas. 52 A

A 25-year-old woman with history of high-grade fever, gradual-onset loss of consciousness and a petechial rash is suspected of having bacterial meningitis. As part of the diagnostic procedure, a lumbar puncture is to be performed. To avoid injury to the spinal cord and nerves you must insert the spinal needle just below the spine of the fourth lumbar vertebra.What anatomical landmark would you use to identify the spine of the fourth lumbar vertebra?

Single best answer question – choose ONE true option onlyPubic symphysis

Iliac crest « YOUR ANSWERXiphoid process

Iliac tuberosity

Umbilicus

YOUR ANSWER WAS CORRECT The Answer

The fourth lumbar vertebra (L4) is a relatively safe level for performing a lumbar puncture. Since the conus medullaris is at the inferior border of L1 or the superior border of L2, it should be safe to insert a needle either above or below L4. The anatomical landmark used to identify L4 is the top of the iliac crest. The line connecting the top of the two iliac crests, the supracrestal line, passes through the spinous process of the L4 vertebra. Therefore, by finding the tops of the iliac crests, you should be able to identify L4.

156

A 29-year-old female underwent total thyroidectomy. Post-operatively she complained of difficulty in breathing.

What would be the most likely cause of this?Unilateral recurrent laryngeal nerve damage during dissection of the upper pole

vessels

Page 119: 1filedownload.com · Web view2014/12/19  · Von-Hippel Lindau disease is a rare inherited pathology that causes cyst development in multiple organs, including the pancreas. 52 A

Hypoparathyroidism in 10% cases

Laryngeal oedema « YOUR ANSWERTachycardia, hyperpyrexia and confusion

Damage to the external superior laryngeal nerve

YOUR ANSWER WAS CORRECT The Answer Comment on this Question

A thyroid crisis should be suspected if there is

tachycardia, hyperpyrexia and confusion.

It is a potentially life-threatening condition that is rarely seen but classically occurs in a patient who has undergone surgery without adequate preoperative preparation.

Assessment of damage to the external superior laryngeal nerve is difficult, as changes are subtle. It is most likely to be damaged at the time of ligation and division of the superior thyroid vessels.

To avoid this, arterial branches should be ligated individually and close to the thyroid and the nerve identified whenever possible. Persistent hypoparathyroidism should be less than 1% in reputable units.

157

Atherosclerosis at the carotid bifurcation is a common and potentially treatable cause of stroke.The right common carotid artery bifurcates at what level?

Angle of mandible

Cricoid cartilage « YOUR ANSWERJugular notch

Upper border of thyroid cartilage « CORRECT ANSWERWithin the substance of the Parotid gland

Page 120: 1filedownload.com · Web view2014/12/19  · Von-Hippel Lindau disease is a rare inherited pathology that causes cyst development in multiple organs, including the pancreas. 52 A

YOUR ANSWER WAS INCORRECT The Answer

The right common carotid artery branches off the brachiocephalic artery. It bifurcates at the level of the upper border of the lamina of the thyroid cartilage. It lies posterior to the lobes of the thyroid gland and anterior to both the cervical sympathetic chain and the phrenic nerve on the scalenus anterior muscle; the latter is separated from the artery by prevertebral fascia.

158

You see a young boy in clinic who has cleft lip and palate.Which of the following is true regarding the incidence and type?

Isolated cleft lip is more common than combined cleft lip and palate deformity

Left cleft lip is more common than bilateral cleft lip « CORRECT ANSWERRight cleft lip is more common than left

The incidence of cleft lip and palate is higher in the African population than in Caucasians

The relative risk of isolated cleft lip or combined cleft lip and palate in a child with one affected sibling is 10%« YOUR ANSWER

YOUR ANSWER WAS INCORRECT The Answer

In Caucasians, cleft lip or cleft palate occurs in approximately 1 in 1000 live births. The incidence is higher in Asia (1 in 500 live births) and lower in Africa (0.4 in 1000 live births).

Cleft lip is thought to be caused by either:failure of fusion between the medial nasal process and the maxillary process; or failure of mesenchymal penetration into the layer between the ectoderm and endoderm.

Combined cleft lip and cleft palate is the most common deformity (45%). Isolated cleft lip occurs in 20% of cases and isolated cleft palate occurs in approximately 30% of cases. Left cleft lip is more common than the right; the incidences of these are

Page 121: 1filedownload.com · Web view2014/12/19  · Von-Hippel Lindau disease is a rare inherited pathology that causes cyst development in multiple organs, including the pancreas. 52 A

higher than bilateral cleft lip (the ratio of left: right: bilateral cleft lip is 6:3:1). The relative risks of a child having isolated cleft lip, or combined cleft lip and palate are:

no history of isolated cleft lip, or combined cleft lip and palate in the family: 0.1% one affected sibling: 4%two affected siblings: 9% one parent and one sibling affected: 17%.

159

A 47-year-old female undergoes a thyroidectomy for Papillary carcinoma of the thyroid gland. During this procedure the inferior thyroid arteries are identified and ligated lateral to the gland.From which vessel does the inferior thyroid artery arise?

Select one answer only

Common Carotid artery

External Carotid artery

Subclavian artery

Thyrocervical trunk « YOUR ANSWERThyroid ima artery

YOUR ANSWER WAS CORRECT The Answer Comment on this Question

The Superior thyroid artery is usually the first branch of the external carotid artery and descends to the gland. In around 10% of people, a thyroid ima artery arises from the brachiocephalic trunk or the arch of the aorta.The inferior thyroid artery is a branch of the thyrocervical trunk, which arises from the subclavian artery.The inferior thyroid artery supplies the inferior pole of the thyroid gland and gives off oesophageal branches. In addition, it supplies both the superior and inferior parathyroid glands. The course of the inferior thyroid artery is closely related to the recurrent laryngeal nerve: in a thyroidectomy, therefore, it is preferable to ligate the artery lateral to the gland to avoid neural damage.

Page 122: 1filedownload.com · Web view2014/12/19  · Von-Hippel Lindau disease is a rare inherited pathology that causes cyst development in multiple organs, including the pancreas. 52 A

160

The supratrochlear nerve is a terminal branch of which nerve?

Single best answer question – choose ONE true option onlyOphthalmic

Frontal « CORRECT ANSWERNasociliary

Lacrimal

Maxillary « YOUR ANSWER

YOUR ANSWER WAS INCORRECT The Answer Comment on this Question

The supratrochlear nerve, the smaller of the two branches of the frontal nerve, passes above the pulley of the superior oblique and gives off a descending filament to join the infratrochlear branch of the nasociliary nerve. It then escapes from the orbit between the pulley of the superior oblique and the supraorbital foramen, curves up onto the forehead close to the bone, ascends beneath the corrugator and frontalis and dividing into branches which pierce these muscles, it supplies the skin of the lower part of the forehead close to the midline and sends filaments to the conjunctiva and skin of the upper eyelid.

161

Skull fractures in the temporal region can damage the middle meningeal artery.

Which of the following best describes this artery?It is a branch of the internal carotid artery

It originates in the infratemporal fossa « YOUR ANSWERIt supplies the pia mater

It is contained within the arachnoid mater

It passes through the foramen ovale

Page 123: 1filedownload.com · Web view2014/12/19  · Von-Hippel Lindau disease is a rare inherited pathology that causes cyst development in multiple organs, including the pancreas. 52 A

YOUR ANSWER WAS CORRECT The Answer

The middle meningeal artery is a branch of the maxillary artery which in turn is a branch of the external carotid artery.

It supplies the dura and calvaria but not the underlying arachnoid mater or pia mater. The middle meningeal artery lies on the bone outside the dura and passes through the foramen spinosum.

The foramen ovale transmits the accessory middle meningeal artery.

162

A motorcyclist who had been involved in a high-speed accident was brought into a major trauma centre intubated and ventilated with full spinal protection. He was unconscious. After examination, he was noted to have priaprism and did not respond to painful stimuli below the clavicle but did respond above the clavicle. The team leader suspected a cervical cord injury. 

What is the most likely reason for this?

Given the mechanism of injury any assessor must be wary of a spinal cord injury

Pre-hospital care teams can cause cervical cord injuries when removing the helmets of injured motorcyclists« YOUR ANSWER

The patient had a blood pressure of 80/50 and a heart rate of 100 BPM

The patient had increased upper body tone

The patient had priapism and grimaced when given a painful stimulus above the clavicle but not below it« CORRECT ANSWER

YOUR ANSWER WAS INCORRECT The Answer

Cervical cord injury is characterised by flaccid areflexia,

Page 124: 1filedownload.com · Web view2014/12/19  · Von-Hippel Lindau disease is a rare inherited pathology that causes cyst development in multiple organs, including the pancreas. 52 A

diaphragmatic breathing and the ability to flex but not extend the elbow.

Priapism is an uncommon but characteristic sign and is said to evolve from the abrupt loss of sympathetic input to the pelvic vasculature leads to increased parasympathetic input and uncontrolled arterial inflow directly into the penile sinusoidal spaces. Full immobilisation of the neck is required at all times and pre-hospital care teams are expert at removing helmets causing minimal movement of the cervical spine. Many motorcycle helmets are fitted with foam inserts, which can be pulled out to assist in their removal.

Because of the loss of sympathetic tone cervical cord injuries often develop neurogenic shock with hypotension as a result of decreased systemic vascular resistance and bradycardia because of unopposed vagal activity. Hypotension without bradycardia in the context of trauma must be assumed to be caused by haemorrhage until proven otherwise. 

The mechanism of injury is associated with spinal column damage and injuries must be ruled out clinically or radiologically.In this case, the combination of priapism with loss of painful stimulus below the clavicle (i.e. dermatome level C5 and below) is more suggestive of cervical cord injury than the other options given.

163

164

165

166

167